Live Test (7th March 2020)

Q1. Which of the following has/have been accorded ‘Geographical Indication’ status?

1. Rasagola 2. Magahi Paan 3. jaggery

Select the correct answer using the code given below: a. 1 only b. 2 and 3 only c. 1 only 3 only d. 1, 2 and 3

Answer: d

Explanation:

Pair (1) is correctly matched: The rasagola, a popular dessert of Odisha, has recently received the geographical indication tag from the Registrar of Geographical Indication. The registration was conferred to ‘Odisha Rasagola’ under Section 16(I) or of authorized Section 17(3)(c) of Geographical Indication of Goods (Registration and Protection) Act 1999. ‘Odisha Rasagola’ is a sweet made of chhena (cottage cheese) cooked in sugar syrup, which is very soft to feel, is juicy and non- chewy in consistency and can be swallowed without teeth pressure.

Pair (2) is correctly matched: Magahi Paan received Geographical Indication Tag in 2018. Magahi paan is a leading cultivar of origin, specially grown in the Magadh regions of four districts-Aurangabad, Gaya, Nawada and Nalanda. Magahi paan growing in these areas are of excellent quality and it is an expensive among other betel leaf. Its betel quid is pungent, less fibrous, and easily soluble inside mouth. It is known for appearance, shiny dark green colour, typical taste, and excellent keeping quality.

Pair (3) is correctly matched:The Marayur jaggery has recently got the geographical indication (GI) tag. It is from the of . Marayur jaggery is produced without adding any chemicals.

Source: https://vajiramias.com/current-affairs/odisha-rasagola/5d412ef01d5def4ee42eb206/

Q2. With respect to “Crime Criminal Tracking Network System (CCTNS) project”, consider the following statements:

1. It was launched in 2001 in the aftermath of Indian Parliament attack to improve policing through e- governance.

2. National Crime Records Bureau (NCRB) is the nodal implementing agency.

Which of the statements given above is/are correct? a. 1 only b. 2 only c. Both 1 and 2 d. Neither 1 nor 2

Answer: b

Explanation:

Statement 1 is incorrect:

The Crime Criminal Tracking Network System (CCTNS) project was launched in 2009 in the aftermath of Mumbai 26/11 attacks. Its objective is to improve policing and criminal justice system through e- governance.

Statement 2 is correct:

Nodal implementing agency is National Crime Records Bureau (NCRB). CCTNS ia launched to computerize Police Processes (FIRs, Investigations, Challans) and inter-link all Police Stations to create a central database by deploying a Core Application Software (CAS) in each state. It has also been decided to implement Interoperable Criminal Justice System (ICJS) which is to be done through integrating CCTNS with e-Courts, e-prisons, Forensics and Prosecution, which are the key components of the Criminal Justice System.

Source: https://vajiramias.com/current-affairs/crime-criminal-tracking-network-system- cctns/5cf6501a1d5def390112779c/

Q3. With reference to the Global Innovation Index-2019 (GII), consider the following statements:

1. It published every year by World Economic Forum (WEF).

2. is the first emerging economy where the GII has been launched.

Which of the statements given above is/are correct? a. 1 only b. 2 only c. Both 1 and 2 d. Neither 1 nor 2

Answer: b

Explanation:

Statement 1 is incorrect:

The GII rankings are published every year by Cornell University, INSEAD and the UN World Intellectual Property Organization (WIPO) and GII Knowledge Partners. The theme of the 2019 GII is Creating Healthy Lives - The Future of Medical Innovation, which aims to explore the role of medical innovation as it shapes the future of healthcare. This is the 12th edition of the GII rankings of 129 economies based on 80 indicators ranging from intellectual property filing rates to mobile-application creation, education spending and scientific and technical publications.

Statement 2 is correct:

Recently Union Minister of Commerce & Industry launched the Global Innovation Index (GII) 2019 in New Delhi. This is the first time that the GII is being launched in an emerging economy.

Switzerland remains number one is the GII index followed by Sweden, the United States of America, the Netherlands, the United Kingdom, Finland, Denmark, Singapore, Germany and Israel. India maintains its top place in the Central and Southern Asia region as the 52nd ranked economy this year. India jumped five places to improve its position from 57th last year to 52nd in 2019. From 81 in 2015, India’s 29-place move up the GII represents the biggest jump by any major economy. Thanks to its high-quality scientific publications and universities, India remains 2nd among middle-income economies in the quality of innovation. This year India reaches the 15th spot in global companies’ R& D expenditures. It also features in the GII ranking on the world’s top science and technology clusters, with Bengaluru, Mumbai and New Delhi included in the global top 100 clusters.

Source: https://vajiramias.com/current-affairs/global-innovation-index- 2019/5d393cf91d5def711f4d1417/

Q4. Which of the following best describes “parthenogenesis”, recently seen in news? a. A security mechanism in computer security for separating running programs, usually in an effort to mitigate system failures. b. It is the farming of aquatic plants and animals in salt water, either in the natural marine environment, or in land- or sea-based enclosures. c. A type of reproduction where the female produces offspring without male fertilization d. It is the growth of an organism in an artificial environment outside the body in which it would normally be found.

Answer: c

Explanation:

Parthenogenesis is defined as “a reproductive strategy that involves development of a female (rarely a male) gamete (sex cell) without fertililisation. A gamete is the egg in females and the sperm in males. In animals, parthenogenesis means development of an embryo from an unfertilized egg cell.

About 2,000 species are known to reproduce through parthenogenesis. It occurs commonly among lower plants and invertebrate animals (particularly rotifers, aphids, ants, wasps and bees) and rarely among higher vertebrates”.

Babies born through parthenogenesis are clones of the mother because there has been no exchange and rearrangement of genetic information with another individual as happens in case of a sexual reproductive process. Hence option (c) is correct.

Source: https://vajiramias.com/current-affairs/parthenogenesis/5cf649201d5def38fd2153d2/ Q5. With respect to “Fugitive Economic Offenders Act, 2018” (FEOA), consider the following statements:

1. Fugitive Economic Offender is an individual who has committed offences involving an amount of 500 crore rupees or more.

2. The order of the special court is final as the appeal against it is not allowed in any court of law.

Which of the statements given above is/are correct? a. 1 only b. 2 only c. Both 1 and 2 d. Neither 1 nor 2

Answer: d

Explanation:

Statement 1 is incorrect:

Fugitive Economic Offender is defined as an individual who has committed offences involving an amount of 100 crore rupees or more and has ran away from India to avoid criminal prosecution.

A special court will be established under the Prevention of Money-laundering Act, 2002 to declare a person as a Fugitive Economic Offender.

Special courts can direct the Central government to seize assets of fugitive economic offender including those that are proceeds of the crime. The Bill allows authorities to provisionally attach properties of an accused, while the application is pending before the Special Court.

Statement 2 is incorrect:

Appeals against the orders of the special court will lie before the High Court.

Source: https://vajiramias.com/current-affairs/fugitive-economic- offender/5cf628821d5def38feab6a39/

Q6. With respect to “COSPAS-SARSAT”, consider the following statements:

1. It is an international satellite-based search and rescue system that uses satellites to detect and locate emergency beacons carried by ships, aircraft, or individuals.

2. It was conceived and initiated by Canada, France, the United States, and the former Soviet Union in 1979.

Which of the statements given above is/are correct? a. 1 only b. 2 only c. Both 1and 2 d. Neither 1 nor 2

Answer: c

Explanation:

Statement 1 is correct:

COSPAS (COsmicheskaya Sisteyama Poiska Avariynich Sudov) is a Russian acronym for Space System for Search of Distress Vessels. SARSAT means Search and Rescue Satellite-Aided Tracking. COSPAS-SARSAT is an international satellite-based search and rescue system that uses satellites to detect and locate emergency beacons carried by ships, aircraft, or individuals. The system consists of a network of satellites, ground stations, mission control centers, and rescue coordination centers. Distress alerts are detected, located and forwarded to over 200 countries and territories at no cost to beacon owners

Statement 2 is correct:

It was conceived and initiated by Canada, France, the United States, and the former Soviet Union in 1979. The definitive agreement of the organization was signed in 1988. It Headquarters is in Montreal, Quebec, Canada.

Source: https://vajiramias.com/current-affairs/cospas-sarsat/5cfcd2e01d5def6808fccc8b/

Q7. Which of the following Stupa recently declared to be a “protected area of national importance” by the Archaeological Survey of India (ASI), which was built to mark the place where Lord Buddha met his first disciples as he traveled from Bodh Gaya to Sarnath? a. Amravati b. Chaukhandi c. Bharhut d. Kesariya

Answer: b

Explanation:

An ancient Buddhist site in ’s Sarnath known as Chaukhandi Stupa has been declared to be a “protected area of national importance” by the Archaeological Survey of India. Hence option (b) is correct.

It is a lofty mound of brick, whose square edifice is surrounded by an octagonal tower. The Chaukhandi Stupa is said to be originally a terraced temple constructed during the Gupta period (4th to 6th Century). Govardhan, the son of Raja Todarmal, gave the present shape to the Chaukhandi Stupa. He built an octagonal tower to commemorate the visit of Humayun, the great Mughal ruler. It was built to mark the place where Lord Buddha met his first disciples as he traveled from Bodh Gaya to Sarnath.

Source: https://vajiramias.com/current-affairs/chaukhandi-stupa/5cfcd2001d5def67fb2f3656/

Q8. With respect to CCR5 protein, recently seen in news, consider the following statements:

1. It is a protein on the surface of red blood cells.

2. It is involved in the immune system as it acts as a receptor for chemokines.

Which of the statements given above is/are correct? a. 1 only b. 2 only c. Both 1 and 2 d. Neither 1 nor 2

Answer: b

Explanation:

Statement 1 is incorrect:

C-C chemokine receptor type 5 is also known as CCR5 or CD195. It is a protein on the surface of white blood cells.

Statement 2 is correct:

It is involved in the immune system as it acts as a receptor for chemokines. A genetic approach involving intrabodies that block CCR5 expression has been proposed as a treatment for HIV-1 infected individuals.The CCR5 gene that encodes the CCR5 protein is located on the short (p) arm at position 21 on chromosome 3.

Source: https://vajiramias.com/current-affairs/ccr5/5cfe13311d5def5ac9c985b9/ Q9. With respect to “Foreigners (Tribunal) Order, 2019”, recently seen in news, consider the following statements:

1. It has empowered district magistrates in all States and UTs to set up Foreigners tribunals to decide whether a person staying illegally in India is a foreigner or not.

2. Only the State administration could move the Tribunal against a suspect.

Which of the statements given above is/are correct? a. 1 only b. 2 only c. Both 1 and 2 d. Neither 1 nor 2

Answer: a

Explanation:

Statement 1 is correct:

Foreigners (Tribunals) Order has empowered district magistrates in all States and UTs to set up Foreigners tribunals to decide whether a person staying illegally in India is a foreigner or not. Earlier, such powers to constitute tribunals vested with the Centre only.

Statement 2 is incorrect:

It has also empowered individuals to approach the Tribunals. Earlier only the State administration could move the Tribunal against a suspect. The amended order also allows District Magistrates to refer individuals who haven’t filed claims against their exclusion from NRC to the Tribunals to decide if they are foreigners or not.

Source: https://vajiramias.com/current-affairs/foreigners-tribunals/5cfe12331d5def5ac485eb5c/

Q10. Principle of ‘significant economic presence’ is related to which of the following? a. SAARC b. BRICS c. IBSA d. OECD

Answer: d

Explanation:

The Organisation for Economic Co-operation and Development (OECD) issued 15 Base Erosion and Profit Shifting (BEPS) Action Plans (APs) in 2015. The first report, BEPS AP 1: Addressing Tax Challenges in the Digital Economy, dealt with tax-related challenges in the digital economy. BEPS AP 1 stated that it is important to examine how enterprises in the digital economy earn profits, since this determines the need to change the existing tax framework to address the specific features in the sector and prevent BEPS. BEPS AP 1 analysed various potential options to address the challenges. These included – The “Equalisation Levy “, which was implemented in India in 2016, and a new economic based nexus in the form of a “Significant Economic Presence (SEP)” which was implemented in India in 2018. Hence option (d) is correct.

The concept of Significant Economic Presence (‘SEP’) was introduced in the Income-Tax Act, 1961 from April 1, 2018 i.e. A.Y. 2019-20. This was introduced to tax the income of the non-resident arising from transactions relating to any goods, services or property in India, including allowing download of data or software or carrying on business activities in India through digital means.

Source: https://vajiramias.com/current-affairs/principle-of-significant-economic-presence- sep/5cfe0fd61d5def5ac9c98580/

Q11. Which of the following straits connects Persian Gulf and the Gulf of Oman? a. Karimata Strait b. Straits of Tiran c. Bab-el-Mandeb d. Strait of Hormuz

Answer: d

Explanation:

The Strait of Hormuz is a strait between the Persian Gulf and the Gulf of Oman. It provides the only sea passage from the Persian Gulf to the open ocean and is one of the world's most strategically important choke points. Hence, option (d) is the correct answer.

Source: https://vajiramias.com/current-affairs/stena-impero/5d3e9b241d5def712229895f/

Q12. With reference to the criteria followed by the Meteorological Department of India (IMD) to classify the low-pressure systems in the Bay of Bengal and in the Arabian Sea, consider the following pairs:

1. Low-Pressure Area: Less than 17 knots (<31 kmph)

2. Depression: 28 to 33 knots (50 to 61 kmph)

3. Cyclonic Storm: 34 to 47 knots (62 to 88 kmph)

4. Severe Cyclonic Storm: 119 knots and above (221 kmph and above)

Select the correct answer using the code given below: a. 1 and 3 only b. 2 and 4 only c. 1 and 3 only d. 1 and 4 only

Answer: c

Explanation:

Recently, ‘VAYU’ was categorised as a Very Severe Cyclonic Storm. The criteria followed by the Meteorological Department of India (IMD) to classify the low-pressure systems in the Bay of Bengal and in the Arabian Sea as adopted by World Meteorological Organisation (WMO) are as under:

• Low-pressure Area: Less than17 knots (<31 kmph) • Depression: 17 to 27 knots (31 to 49 kmph) • Deep Depression: 28 to 33 knots (50 to 61 kmph) • Cyclonic Storm: 34 to 47 knots (62 to 88 kmph) • -Severe Cyclonic Storm: 48 to 63 knots (89 to 118 kmph) • Very Severe Cyclonic Storm: 64 to 119 knots (119 to 221 kmph) • -Super Cyclonic Storm: 119 knots and above (221 kmph and above)

Hence option (c) is correct.

Source: https://vajiramias.com/current-affairs/classification-of-tropical- cyclones/5d033abd1d5def0e116bfc8f/

Q13. The World Investment Report 2019 was released by which of the following institutions? a. World Bank b. UN Conference on Trade and Development (UNCTAD) c. World Economic Forum d. Organisation for Economic Co-operation and Development

Answer: b

Explanation:

The World Investment Report 2019 was released by the UN Conference on Trade and Development (UNCTAD). Hence option (b) is correct.

Global FDI inflows have been facing headwinds for the last few years. As per UNCTAD’s World Investment Report 2019, Global Foreign Direct Investment (FDI) flows slid by 13% in 2018 to the US $1.3 trillion in the previous year, that is the third consecutive annual decline. Despite the dim global picture, India continues to remain a preferred and attractive destination for Global FDI flows. However, it is felt that the country has the potential to attract far more Foreign Investment which can be achieved, inter- alia, by further liberalizing and simplifying the FDI policy regime.

Key findings of the report are Foreign Direct Investment (FDI) to India grew by 6 % to 42 billion dollars in 2018, with strong inflows in the manufacturing, communication and financial services sectors, and cross- border merger and acquisition activities.India was among the top 20 host economies for FDI inflows in 2017-18. FDI inflows to South Asia increased by 3.5 % to 54 billion dollars. India has historically accounted for 70 to 80 % of inflows to the sub-region.

Source: https://vajiramias.com/current-affairs/world-investment-report- 2019/5d0343321d5def0e0aea560e/

Q14. With respect to “Asiatic golden cat”, consider the following statements:

1. It is only the golden colour to which it is be associated. 2. It is listed as endangered on the International Union for Conservation of Nature’s (IUCN) Red List of threatened species. Which of the statements given above is/are correct? a. 1 only b. 2 only c. Both 1 and 2 d. Neither 1 nor 2

Answer: d

Explanation:

Statement 1 is incorrect:

Scientists have discovered that Golden is no longer the only colour the elusive Asiatic golden cat can be associated with. Its coat comes in five other shades in Arunachal Pradesh. Scientists have discovered six different 'colour morphs' of the Asiatic golden cat in Arunachal Pradesh, that may be the world’s greatest number of different-coloured wild cat species ever reported in one area.

Statement 2 is incorrect:

Asiatic golden cat (Catopuma temminckii) is listed as near threatened on the International Union for Conservation of Nature’s Red List of threatened species.It is found across eastern Nepal through north- eastern India to Indonesia. Bhutan and China were known to have two morphs of the golden cat — one the colour of cinnamon and the other with markings similar to the ocelot, a small wild cat found in the Americas.

Source: https://vajiramias.com/current-affairs/asiatic-golden-cat/5d0351321d5def0e0752ee09/

Q15. With reference to the New Delhi International Arbitration Centre Act 2019, consider the following statements:

1. Its objective is to create an independent anda autonomous regime for only international arbitration in India. 2. It will be headed by a Chairperson, who has been a retired Chief Justice of India.

Which of the statements given above is/are correct? a. 1 only b. 2 only c. Both 1 and 2 d. Neither 1 nor 2

Answer: b

Explanation:

Statement 1 is incorrect: New Delhi International Arbitration Centre Act, 2019 aims to create New Delhi International Arbitration Centre as an independent and autonomous regime for institutionalized domestic and international arbitration and establishing India as an International Hub of Arbitration.

Statement 2 is correct:

The NDIAC will be headed by a Chairperson, who has been a Judge of the Supreme Court or a Judge of a High Court or an eminent person, having experience in arbitration to be appointed by the Central Government in consultation with the Chief Justice of India. It will also have two Full-time or Part-time Members from amongst eminent persons having experience in arbitration. In addition, one representative of a recognized body of commerce and industry shall be nominated on a rotational basis as a Part-time Member. The Secretary, Department of Legal Affairs, Ministry of Law & Justice, Financial Adviser nominated by Department of Expenditure, Ministry of Finance and Chief Executive Officer, NDIAC will be ex-officio Members.

Source: https://vajiramias.com/current-affairs/new-delhi-international-arbitration-centre- ndiac/5d032ff51d5def0e0bcf7493/

Q16. Which of the following sites was the “largest cemetery of the Late Harappan period of the early 2nd millennium BCE”? a. Daimabad, b. Sadikpur Sinauli, Uttar Pradesh c. Khargone, d. Kurnool,

Answer: b

Explanation:

The ancient site at Sadikpur Sinauli is spread over 28 hectares in Uttar Pradesh’s Baghpat district. The site was the “largest cemetery (necropolis) of the late Harappan period of the early 2nd millennium BCE”. Hence option (b) is correct.

The burials are all oriented in a NW-SE direction and most are identified as primary burials. Some of the burials are identified as secondary, multiple and symbolic burials.

At the site, the excavation and preservation work is still ongoing after being started in 2018 and was deemed to have national importance due to the finds uncovered. Among the treasures unearthed are three chariots, legged coffins, shields, swords and helmets – all which point towards a warrior class that must have existed around 2,000 BCE.

Source: https://vajiramias.com/current-affairs/sadikpur-sinauli/5d0c98a51d5def0e1251869f/

Q17. With respect to “Carbon Quantum Dots (CQDs)”, consider the following statements:

1. They are carbon-based nanomaterials whose size is less than 10 nm, or nanometre. 2. They are not only used as diagnostic tools for bio-imaging but also for chemical sensing and in optoelectronics.

Which of the statements given above is/are correct? a. 1 only b. 2 only c. Both 1 and 2 d. Neither 1 nor 2

Answer: c

Explanation:

Statement 1 is correct:

Carbon Quantum Dots (CQDs) are carbon-based nanomaterials whose size is less than 10 nm, or nanometre.

Statement 2is correct:

As an emerging class of luminescent nanomaterials, carbon quantum dots (CQDs) have recently shown enormous potential for imaging applications on account of their characteristic broad emission, tunable fluorescence emission, high thermal stability, and low cytotoxicity. They are used as diagnostic tools for bio-imaging, especially in detecting cancer cells, for chemical sensing and in optoelectronics.

The team of scientists from CSIR-NEIST in have recently developed a chemical method of producing CQDs from cheap, abundant, low-quality and high-sulphur coals.

Source:https://vajiramias.com/current-affairs/carbon-quantum-dots- cqds/5d0c99701d5def0e0bd03713/

Q18. With respect to the “Strategic Partnership Model”, consider the following statements:

1. It was approved by the Defence Ministry in 2017 as a part of the Defence Procurement Procedure (DPP). 2. The construction of six conventional submarines for P-75(I) Project of the Indian Navy is the first project being undertaken under the latest Strategic Partnership Model.

Which of the statements given above is/are correct? a. 1 only b. 2 only c. Both 1 and 2 d. Neither 1 nor 2

Answer: a Explanation:

Statement 1 is correct: Strategic Partnership (SP) policy was approved by the Defence Ministry in May 2017 as a part of the Defence Procurement Procedure (DPP). In July 2018, the Defence Acquisition Council (DAC) approved Implementation guidelines for the SP Model. Its objective is to reduce current dependence on imports and to bolster defence manufacturing in India through indigenous private defence firms.

Statement 2 is incorrect: P-75(I) Project of the Indian Navy is the second project being undertaken under the latest Strategic Partnership Model, with the first being the procurement of 111 Naval Utility Helicopters.

Source: https://vajiramias.com/current-affairs/strategic-partnership- model/5d0c9a751d5def0e125186b6/

Q19. ‘Garuda-VI’ is a bilateral exercise between the Indian Air Force and which of the following countries? a. Russia b. US c. France d. Indonesia

Answer: c

Explanation:

Exercise Garuda-VI is a bilateral exercise between the Indian and French Air Force. This is the sixth edition of Exercise Garuda and is being planned at French Air Force Base, Mont- de-Marsan. The last exercise, ‘Garuda V’ was held at Air Force Station Jodhpur in June 2014. Hence option (c) is correct.

Source: https://vajiramias.com/current-affairs/garuda-vi/5d1307131d5def0e0aebb7cc/

Q20. With respect to “Elephant Bonds”, recently seen in the news, consider the following statements:

1. It is a specialised security product providing funds towards Long Term Infrastructure. 2. It is recommended by Surjit Bhalla Committee constituted by the Department of Commerce.

Which of the statements given above is/are correct? a. 1 only b. 2 only c. Both 1 and 2 d. Neither 1 nor 2

Answer: c

Explanation: A High-Level Advisory Group (HLAG) on Trade and Policy, headed by Dr. Surjit S. Bhalla has been constituted by the Department of Commerce.

Recommendations:

• The HLAG has made several recommendations for boosting India’s share and importance in global merchandise and services trade. • It recommended “Elephant Bonds” as a specialised security product providing funds towards Long Term Infrastructure. This bond is issued to those people who declare their previously undisclosed income and are then bound to invest 50% of that amount in these securities. The fund gathered by the issuance of these bonds is utilized to finance infrastructure projects only. A high-level trade panel has estimated that India could recover up to $500 billion of black money stashed overseas if it implements its proposal to issue these bonds because such bonds would be an avenue for people to bring their offshore undisclosed wealth into India without fear of prosecution. • It identifies tax reforms to boost export and investment channels for exports. • It also made recommendations for reforms in Financial Services Framework for making India a Preferred Destination for financial services.

Hence both the statements are correct.

Source: https://vajiramias.com/current-affairs/surjit-bhalla-committee/5d1306c41d5def0e0754488d/

Q21. With respect to “Proton Therapy”, consider the following statements:

1. It uses protons rather than x-rays to treat cancer. 2. It is a type of external-beam radiation therapy.

Which of the statements given above is/are correct? a. 1 only b. 2 only c. Both 1 and 2 d. Neither 1 nor 2

Answer: c

Explanation:

Statement 1 is correct:

Proton therapy is a type of Radiation therapy which is also called proton beam therapy. It uses protons rather than x-rays to treat cancer. At high energy, protons can destroy cancer cells. It can also be combined with x-ray radiation therapy, surgery, chemotherapy, and/or immunotherapy.

Statement 2 is correct:

Like x-ray radiation, proton therapy is a type of external-beam radiation therapy. External beam radiation therapy comes from a machine that aims radiation at your cancer. It is a local treatment, which means it treats a specific part of your body. For example, if you have cancer in your lung, you will have radiation only to your chest, not to your whole body. External beam radiation therapy is used to treat many types of cancer. Radiation beams used in external radiation therapy come from three types of particles:

• Photons – Most radiation therapy machines use photon beams. Photons are also used in x-rays, but x-rays use lower doses. Photon beams can reach tumours deep in the body. As they travel through the body, photon beams scatter little bits of radiation along their path. These beams do not stop once they reach the tumour, but go into normal tissue past it. • Protons – Protons are particles with a positive charge. Proton beams can also reach tumours deep in the body. However, proton beams do not scatter radiation on their path through the body and they stop once they reach the tumour. Doctors think that proton beams might reduce the amount of normal tissue that is exposed to radiation. Clinical trials are underway to compare radiation therapy using proton beams with that using photons beams. Some cancer centres are using proton beams in radiation therapy, but the high cost and size of the machines are limiting their use. • Electrons – Electrons are particles with a negative charge. Electron beams cannot travel very far through body tissues. Therefore, their use is limited to tumours on the skin or near the surface of the body. In India, Tata Memorial Centre (TMC), a grant-in-aid Institution under Department of Atomic Energy, has initiated a collaboration with Bhabha Atomic Research Centre (BARC) and Tata Institute of Fundamental Research (TIFR) for research in proton therapy.

Source: https://vajiramias.com/current-affairs/proton-therapy/5d130aa21d5def0e07544902/

Q22. With respect to “Impressed Tortoise”, consider the following statements:

1. For the first time wildlife experts have discovered the Impressed Tortoise in Arunachal Pradesh. 2. It is categorized as critically endangered in IUCN Red List Status.

Which of the statements given above is/are correct? a. 1 only b. 2 only c. Both 1 and 2 d. Neither 1 nor 2

Answer: a

Explanation:

Statement 1 is correct:

Wildlife experts have recently discovered the Impressed Tortoise in Arunachal Pradesh. This species of tortoise has never been previously sighted in India.

Statement 2 is incorrect: Impressed Tortoise (Manouria Impressa) is categorized as vulnerable in IUCN Red List Status.It occurs in mountainous forest areas in Southeast Asia in Burma, southern China, Thailand, Laos, Vietnam, Cambodia, Malaysia and now has been found in India. The species has a golden-brown shell and skin. India was known to be the home of only the Asian Forest Tortoise (Manouria emys) until the discovery of the Impressed Tortoise. The Asian Forest Tortoise, the largest in mainland Asia, is found only in the northeast, as are 20 of the other 28 species of chelonians.

Source:https://vajiramias.com/current-affairs/impressed-tortoise/5d13089f1d5def0e0f164b9d/

Q23. Which of the following are not eligible for PM Kisan Samman Nidhi Scheme?

1. Multi-Tasking Staff / Class IV/Group D employees. 2. All Persons who paid Income Tax in the last assessment year. 3. All non-institutional landholder farmers’ families in the country. 4. Mayors of Municipal Corporations and Chairpersons of District Panchayats.

Select the correct answer using the code given below: a. 1, 2 and 4 only b. 2 and 4 only c. 1 and 3 only d. 1, 2, 3 and 4

Answer: b

Explanation:

All landholder farmers’ families in the country are eligible for the PM-Kisan Scheme, except those covered under the following exclusion criteria:

• All Institutional Landholders; and • Farmer families in which one or more of its members belong to the following categories: • Former and present holders of constitutional posts. • Former and present Ministers, Members of Parliament & state legislature, Mayors of Municipal Corporations and Chairpersons of District Panchayats. • All serving or retired employees of Central/ State Government Ministries, PSEs and as well as regular employees of the Local Bodies (Excluding Multi-Tasking Staff / Class IV/Group D employees). • All superannuated/retired pensioners whose monthly pension is Rs.10,000/-or more (Excluding Multi-Tasking Staff / Class IV/Group D employees). • All Persons who paid Income Tax in the last assessment year. • Professionals like Doctors, Engineers, Lawyers, Chartered Accountants etc. Hence option (b) is correct.

Source: https://vajiramias.com/current-affairs/eligibility-for-pm-kisan-samman-nidhi- scheme/5d1307891d5def0e12521744/ Q24. Consider the following statements regarding rocket fuels and propulsion:

1. ASCENT green fuel is a safer rocket fuel which will not use hydroxyl ammonium nitrate fuel/oxidizer blend. 2. LightSail 2 is the first orbiting spacecraft to be propelled solely by artificial light.

Which of the statements given above is/are correct? a. 1 only b. 2 only c. Both 1 and 2 d. Neither 1 nor 2

Answer: d

Explanation:

Statement 1 is incorrect:

Recently SpaceX launched its Falcon Heavy spacecraft on its third mission, comprising huge payload – 24 satellites from various organisations, including government agencies – including the Deep Space Atomic Clock (DSAC) and ASCENT green fuel. One of the satellites will be a test spacecraft for safer rocket fuel. The traditional fuel used in satellites is hydrazine, which is extremely toxic to humans as well as the environment.The new alternative is called ASCENT (Advanced Spacecraft Energetic Non-toxic Propellant), which is a hydroxyl ammonium nitrate fuel/oxidiser blend.ASCENT is described as a fuel with significantly reduced toxicity levels compared to hydrazine, and potentially shorter launch processing times, resulting in lower costs.

Statement 2 is incorrect:

LightSail 2 is a crowd-funded solar sail project from the Planetary Society.It seeks to become the first orbiting spacecraft to be propelled solely by sunlight.

Source: https://vajiramias.com/current-affairs/deep-space-atomic-clock-ascent-green- fuel/5d1309b71d5def0e075448e6/

Q25. ‘75 Student Satellites Mission 2022’, is an initiative of which of the following organizations? a. Indian Technology Congress Association (ITCA) b. Indian Space Research Organisation (ISRO) c. Defence Research and Development Organisation (DRDO) d. University Grants Commission of India (UGC)

Answer: a

Explanation: The Indian Technology Congress Association (ITCA) has a project titled Seventy- five by 75 to celebrate the nation’s 75th birthday. Under its ‘75 Student Satellites Mission 2022’, up to 75 tiny satellites built by students of Indian universities could fly to the skies between late next year and 2022 in batches.

Students of participating institutions would come from different disciplines and get to build nanosatellites weighing between 3 and 12 kg. They may demonstrate a novel concept, science experiment, or technology in orbit. Assembling a satellite of their own, creating its ground control system, and operating the spacecraft would be a creative experience and give them an edge in the job market

The ITCA, a technology promotion body based in Bengaluru, has roped in around 40 engineering colleges to form a consortium. It also has launch contracts with the Indian Space Research Organisation and Israeli finance for its ‘75 Student Satellites Mission 2022’,

Hence, option (a) is the correct answer.

Source: https://vajiramias.com/current-affairs/75-student-satellites-mission- 2022/5d15bae31d5def7fd16c9be4/

Q26. What is ‘L 98-59b’, which was in the news recently? a. Exoplanet b. Advanced Air Defence (AAD) Missile of Russia c. An asteroid of the Apollo group d. None of the above

Answer: a

Explanation:

NASA’s Transiting Exoplanet Survey Satellite (TESS) has discovered a new planet named L 98-59b. L 98- 59b is around 80% of Earth’s size. It is between the sizes of Mars and Earth. It is the tiniest of Transiting Exoplanet Survey Satellite (TESS) finds so far. It orbits a bright, cool, nearby star named L 98-59. The host star, is about one-third the mass of the Sun and lies about 35 light-years away. Hence, option (a) is the correct answer.

NASA’s Transiting Exoplanet Survey Satellite (TESS) is an all-sky survey mission that will discover thousands of exoplanets around nearby bright stars. TESS was launched in April 2018 aboard a SpaceX Falcon 9 rocket.

Source: https://vajiramias.com/current-affairs/l-98-59b/5d15c7ab1d5def7fc5001f99/

Exoplanet Detection All of the planets in our solar system orbit around the Sun. Planets that orbit around other stars are called exoplanets. Exoplanets are very hard to see directly with telescopes. They are hidden by the bright glare of the stars they orbit. So, astronomers use other ways to detect and study these distant planets. They search for exoplanets by looking at the effects these planets have on the stars they orbit. • One way to search for exoplanets is to look for "wobbly" stars. A star that has planets doesn’t orbit perfectly around its centre. From far away, this off-centre orbit makes the star look like it’s wobbling. Hundreds of planets have been discovered using this method. However, only big planets—like Jupiter, or even larger—can be seen this way. Smaller Earth-like planets are much harder to find because they create only small wobbles that are hard to detect. • In 2009, NASA launched a spacecraft called Kepler to look for exoplanets. Kepler looked for planets in a wide range of sizes and orbits. And these planets orbited around stars that varied in size and temperature. Some of the planets discovered by Kepler are rocky planets that are at a very special distance from their star. This sweet spot is called the habitable zone, where life might be possible. Kepler detected exoplanets using something called the transit method. When a planet passes in front of its star, it’s called a transit. As the planet transits in front of the star, it blocks out a little bit of the star's light. That means a star will look a little less bright when the planet passes in front of it. Astronomers can observe how the brightness of the star changes during a transit. This can help them figure out the size of the planet. So far, thousands of planets have been discovered by the Kepler mission. And more will be found by NASA's Transiting Exoplanet Survey Satellite (TESS) mission, which is observing the entire sky to locate planets orbiting the nearest and brightest stars.

Q27. With respect to “Elephanta Caves”, consider the following statements:

1. It consist of group of caves belongs to Buddhism, Jainism and Hinduism 2. it’s a UNESCO World Heritage Site.

Which of the statements given above is/are correct? a. 1 only b. 2 only c. Both1 and 2 d. Neither 1 nor 2

Answer: b

Explanation:

Statement 1 is incorrect:

Elephanta Caves are located in Mumbai Harbour, on the Elephant island, also known as Gharapuri Island. The origins and date when the caves were constructed have attracted considerable speculations and scholarly attention. Most scholars consider it to have been completed by about 550 CE.

The island consists of two groups of caves: (1) Hindu cave temples which contain rock cut stone sculptures dedicated mainly to Lord Shiva and (2) Buddhist caves. However there are no Jaina caves on the island.

Statement 2 is correct:

It’s a UNESCO World Heritage Site.Caves were cut into granite rocks.The layout of the caves, including the pillar components, the placement and division of the caves into different parts, and the provision of a sanctum or Garbhagriha of sarvatobhadra plan, are important developments in rock-cut architecture. They are called Elephanta caves because the Portuguese called the island Elephanta on seeing its huge gigantic statue of an Elephant at the entrance.

Source: https://vajiramias.com/current-affairs/elephanta-festival-of-art-and- culture/5cf4e0341d5def1dc45e7e81/

Q28. What is “Utkarsh 2022”, recently seen in the news? a. It is the global conservation initiative to focus specifically on threatened languages and art forms that represent a significant amount of unique evolutionary history. b. It is a three-year road map for medium-term objective to be achieved for improving regulation, supervision of the RBI. c. It is an ambitious project of India to send a mission to Mars. d. It is an initiative to bridge the digital divide and accelerate development on the African continent.

Answer: b

Explanation:

Utkarsh 2022 is a three-year road map for medium-term objective to be achieved for improving regulation, supervision of the RBI. This medium-term strategy is in line with the global central banks’ plan to strengthen the regulatory and supervisory mechanism. An internal committee was formed to identify issues that needed to be addressed over the next three years. Hence option (b) is correct.

Source: https://vajiramias.com/current-affairs/utkarsh-2022/5d24625b1d5def5a860f5db9/

Q29. With respect to “NAG missile”, consider the following statements:

1. It is a third-Generation fire-and-forget class Anti-Tank Guided Missile. 2. It has been developed to engage in all weather conditions with both day and night capabilities.

Which of the statements given above is/are correct? a. 1 only b. 2 only c. Both 1 and 2 d. Neither 1 nor 2

Answer: c

Explanation:

Statement 1 is correct:

It is a third-generation fire and forget class missile and uses an imaging infrared seeker in lock-on- before-launch mode. The missile is launched from the NAG missile carrier (NAMICA) which is capable of carrying up to six combat missiles. Statement 2 is correct:

NAG is a third-generation Anti-Tank Guided Missile. It is developed by Defence Research and Development Organisation (DRDO). NAG missile has been developed to engage highly fortified enemy tanks in all weather conditions with day and night capabilities and with a minimum range of 500 metres and maximum range of 4 kilometres.

Source: https://vajiramias.com/current-affairs/nag-missile/5d328c7e1d5def45da1888a7/

Q30. Which of the following is a narrow strait that connects the Atlantic Ocean to the Mediterranean Sea? a. Strait of Gibraltar b. Sunda Strait c. Bosphorus Strait d. Strait of Makassar

Answer: a

Explanation:

Gibraltar is a British Overseas Territory. It is located at the southern tip of the Iberian Peninsula. it is bordered to the north by Spain. It is a narrow strait that connects the Atlantic Ocean to the Mediterranean Sea and separates Gibraltar and Peninsular Spain in Europe from Morocco and Ceuta (Spain) in Africa. The landscape of Gibraltar is dominated by the Rock of Gibraltar (made of Jurassic limestone) at the foot of which is a densely populated town area. Hence, option (a) is the correct answer.

In 1704, Anglo-Dutch forces captured Gibraltar from Spain during the War of the Spanish Succession on behalf of the Habsburg claim to the Spanish throne. The territory was ceded to Great Britain in perpetuity under the Treaty of Utrecht in 1713.

Source: https://vajiramias.com/current-affairs/gibraltar/5d32be221d5def5049d8d7e6/

Q31. What is Pegasus, recently seen in the news? a. It is Israeli spyware that can be installed on smartphones. b. It is the second-closest known exoplanet to our Sun. c. It is an Advanced Air Defence (AAD) Missile of USA d. None of the above

Answer: a

Explanation:

Pegasus is a spyware that can be installed on mobile devices. It is regarded as one of the ‘most sophisticated’; smartphone spyware. Upon clicking on a malicious link on the device, Pegasus is capable of reading text messages, tracking calls, collecting passwords, tracing the location of the phone, accessing the target device’s microphone and gathering information from apps. It has been developed by the Israeli cyber arms firm, NSO Group. However, NSO says it does not operate the Pegasus system, only licensing it to closely vetted government users “for the sole purpose of preventing or investigating serious crime including terrorism”. Hence, option (a) is the correct answer.

Source: https://vajiramias.com/current-affairs/pegasus/5d32b05a1d5def50421f9c86/

Q32. DASTAK campaign which was in news recently, aims to: a. Eradicate Acute Encephalitis Syndrome (AES) and Japanese Encephalitis (JE) disease. b. Eradicate Polio disease. c. Protect Elephant corridors d. None of the above

Answer: a

Explanation:

Uttar Pradesh government recently launched the DASTAK campaign to eradicate deadly Acute Encephalitis Syndrome (AES) and Japanese Encephalitis (JE) disease. The campaign will run from 1st to 31st July. Under this state-wide campaign, teams will go door to door in every village of the 75 districts of the state to create awareness about the communicable diseases as well as Acute Encephalitis Syndrome (AES) and Japanese Encephalitis (JE). Hence, option (a) is the correct answer.

Source: https://vajiramias.com/current-affairs/dastak-campaign/5d19a9ca1d5def197a99493f/

Q33. With reference to the International Security Alliance (ISA), consider the following statements:

1. Qatar is hosting ISA’s first joint security exercise 2. ISA is an international working group to confront organised, transnational and extremist crimes.

Which of the statements given above is/are correct? a. 1 only b. 2 only c. Both 1 and 2 d. Neither 1 nor 2

Answer: b

Explanation:

Statement 1 is incorrect:

Abu Dhabi is hosting the International Security Alliance (ISAs) first joint security exercise named ISALEX19. Representatives of 50 law enforcement agencies of the International Security Alliance are taking part in the first joint security exercise in Abu Dhabi. The participants include representatives from tactical teams, rapid intervention units, communications, civil defence, and explosive ordnance disposal, teams.

Statement 2 is correct:

The International Security Alliance (ISA) is an international working group to confront organised, transnational and extremist crimes. It was launched in 2017 in Abu Dhabi. The alliance now comprises nine countries, the United Arab Emirates, Bahrain, Morocco, France, Italy, Spain, Senegal, Singapore, and the Slovak Republic.

Source: https://vajiramias.com/current-affairs/isalex19/5d19a8901d5def196fe3dc2a/

Q34. Which of the following is the highest peak in North America? a. Mount Denali b. Mount Lhotse c. Mount Aconcagua d. Mount Kilimanjaro

Answer: a

Explanation:

Indo-Tibetan Border Police (ITBP) Deputy Inspector General (DIG) Aparna Kumar has become the first civil servant to scale Mount Denali, the highest peak in North America.

Mount Denali was once called Mount McKinley. It was officially named Denali after protests by the native Koyukon Athabascan people who call the mountain Denali, which is usually translated as "The Great One”.

It is located in south-central Alaska. The mountain's peak is 20,310 feet (6,190 meters) above sea level, thus making it the –tallest mountain in North America and the third highest mountain of the Seven Summits — the highest mountains on each of the seven continents — following Mount Everest in Nepal and Aconcagua in Argentina. Hence, option (a) is the correct answer.

Source: https://vajiramias.com/current-affairs/mount-denali/5d19a8fc1d5def197a99492f/

Q35. Who among the following arrreee eligible to get free legal services under the Legal Services Authorities Act?

1. Small marginal farmer 2. Victim of human trafficking 3. Differently-abled person 4. Industrial workman

Select the correct answer using the code given below: a. 2 and 3 only b. 2, 3 and 4 only c. 1, 2 and 3 only d. 1, 2, 3 and 4

Answer: b

Explanation:

In 1987, the Legal Services Authorities (LSA) Act was enacted to give free and competent legal services to the poor. The Act paved the way for the constitution of National Legal Service Authority (NALSA) and other legal service institutions at the State, district and taluka level. Free legal services under LSA Act are available to a person belonging to Schedule Tribe and Schedule Caste, woman, child, victim of human trafficking, differently-abled person, industrial workman, and person in custody in a protective home and the poor. Hence option (b) is correct.

Source: https://vajiramias.com/current-affairs/free-legal-aid/5d1b20bb1d5def5a80bad63c/

Q36. Which of the following pairs is/are correctly matched?

Dams States

1. Tiware Kerala

2. Bhakra Odisha

3. Tehri

Select the correct answer using the code given below: a. 1 only b. 1 and 2 only c. 2 and 3 only d. None of the above Answer: d

Explanation:

Pair (1) is not correctly matched: At least six people were killed and more than 20 are reportedly missing after Tiware Dam in Maharashtra breached following heavy rain. The Tiware dam is located on a rivulet in Chiplun taluka of Ratnagiri district in Maharashtra. The Tiware dam had started developing cracks after Tuesday evening. Before villagers downstream could be alerted, the dam developed a rupture flooding the villages and hamlets within minutes.

Pair (2) is not correctly matched: Bhakra Dam is a concrete gravity dam on the Sutlej River in Bilaspur, Himachal Pradesh in northern India. The dam forms the Gobind Sagar reservoir.

Pair (3) is not correctly matched: The Tehri Dam is the highest dam in India and one of the highest in the world. It is a multi-purpose rock and earth-fill embankment dam on the Bhagirathi River near Tehri in , India.

Source: https://vajiramias.com/current-affairs/tiware-dam/5d1c60f21d5def5a7ceaf155/

Q37. Which of the following are the Kharif crops in India?

1. Maize 2. Sorghum 3. Chickpea 4. Wheat

Select the correct answer using the code given below: a. 1 and 4 only b. 1 and 2 only c. 1, 2 and 3 only d. 1, 2, 3 and 4

Answer: b

Explanation:

The agricultural crop year in India is from July to June. The Indian cropping season is classified into two main seasons-(i) Kharif and (ii) Rabi based on the monsoon.The kharif cropping season is from July – October during the south-west monsoon and the Rabi cropping season is from October-March (winter). The crops grown between March and June are summer crops. Pakistan and Bangladesh are two other countries that are using the term ‘kharif and ‘rabi’ to describe about their cropping patterns. The terms ‘kharif’ and ‘rabi’ originate from Arabic language where Kharif means autumn and Rabi means spring.The kharif crops include rice, maize, sorghum, pearl millet/bajra, finger millet/ragi (cereals), arhar (pulses), soyabean, groundnut (oilseeds), cotton etc. The rabi crops include wheat, barley, oats (cereals), chickpea/gram (pulses), linseed, mustard (oilseeds) etc. Hence, option (b) is the correct answer.

Source: https://vajiramias.com/current-affairs/msp-of-kharif-crops/5d1d847f1d5def5a7ec38b6a/ Q38. Which of the following pairs is/are correctly matched?

Region (often in news) Country 1. Kulhudhuffushi Sri Lanka 2. Puerto Williams Brazil

3. Kobe China

Select the correct answer using the code given below: a. 1, 2 and 3 b. 1 and 2 only c. 2 and 3 only d. None of the above

Answer: d

Explanation:

Pair (1) is not correctly matched: The Union Cabinet has given ex-post facto approval for the MoU between India and Maldives for the establishment of passenger and cargo services by sea. The MoU will pave way for ferry services between Maldives and Kerala by Connecting with Male & Kulhudhuffushi in Maldives by Sea route. While Male – the capital and most populous city – is situated at a distance of 708 Kms from Kochi, Kulhudhuffushi – the third most populous city of Maldives – is 509 Kms away.

Pair (2) is not correctly matched: Puerto Williams is a town on Navarino Island in the Beagle Channel in Chile’s far south. It is now the world’s southernmost city. Puerto Williams has nudged out Ushuaia, a neighbour in Argentina’s Tierra del Fuego, for the title.

Pair (3) is not correctly matched: Authorities from the Japanese city of Kobe recently exchanged a Letter of Intent (LoI) with their counterparts in Ahmedabad for a sister city partnership, which will pave the way for an enhanced economic relationship between the two vibrant cities as well as the two countries. In November 2016, PM of India and his Japanese counterpart Shinzo Abe inked a sister-state relationship MoU for and Hyogo prefecture. Kobe is the capital city of Hyogo. The MoU sought to promote mutual cooperation between Gujarat and Hyogo in the fields of academics, business, cultural cooperation, disaster management and environmental protection.

Source: https://vajiramias.com/current-affairs/kulhudhuffushi/5d1d83f01d5def5a7ec38b52/

Q39. Which of the following is known as the Kutchi New Year? a. Ashadhi Bij b. Ugadi c. Pahela Baishakh d.

Answer: a

Explanation:

Ashadi Beej or Ashadhi Bij is observed on the second day of the Shukla Paksha or waxing phase of moon in Ashada month. It is the Kutchi New Year. This Hindu New Year is observed in the Kutch region in Gujarat. Ashadi Beej is mostly a traditional affair and is a celebration of rains. During Ashadhi-beej, farmers check the moisture in the atmosphere to help predict which crop would do best incoming monsoon. Hence, option (a) is the correct answer.

Source: https://vajiramias.com/current-affairs/ashadhi-bij/5d1ee8811d5def5a7ceb35f0/

Q40. La Silla Observatory is an astronomical observatory located in which of the following places? a. Gobi Desert b. c. Atacama Desert d. Arizona-Sonoran Desert

Answer: c

Explanation:

La Silla Observatory is an astronomical observatory in Chile located at the outskirts of the Chilean Atacama Desert, one of the driest and most remote areas of the world. It has three telescopes built and operated by the European Southern Observatory (ESO). The observatory is one of the largest in the Southern Hemisphere and was the first in Chile to be used by ESO. Hence, option (c) is the correct answer.

Source: https://vajiramias.com/current-affairs/great-south-american- eclipse/5d1eea921d5def5a84ea4408/

Q41. Which of the following countries recently introduced a new Digital 'GAFA Tax'? a. France b. China c. Israel d. India

Answer: a

Explanation:

In France, the lower house of parliament approved a small, pioneering tax on internet giants like Google, Amazon and Facebook. The so-called GAFA tax has a French acronym that indicates it was inspired by US giants such as Google, Apple, Facebook and Amazon, and targets digital companies with global annual sales of more than 750 million euros ($849 million) and sales in France of at least 25 million euros. Hence, option (a) is the correct answer.

At present, digital companies pay nearly no tax in countries where they have large sales like France. The bill aims to stop multinationals from avoiding taxes by setting up headquarters in low-tax EU countries. The bill foresees a 3% tax on the French revenues of digital companies with global revenue of more than 750 million euros ($847 million), and French revenue over 25 million euros.

Source: https://vajiramias.com/current-affairs/tax-on-online-giants/5d1ee9db1d5def5a7ceb3601/

Q42. Which of the following best describes the objective of MOSAIC mission that was in news recently? a. Studying the impact of climate change on the Arctic and how it could affect the rest of the world. b. Study Saturn's moon Titan for Origins and Signs of Life. c. Studying Mars' interior structure to know about the early formation of rocky planets in our inner solar system. d. To reveal the story of Jupiter's formation and evolution.

Answer: a

Explanation:

The MOSAiC mission stands for Multidisciplinary drifting Observatory for the Study of Arctic Climate. The mission objective is studying the impact of climate change on the Arctic and how it could affect the rest of the world. It will be a year-long mission. The mission has received funding from U.S. institutions such as the National Science Foundation, the Department of Energy, the National Oceanic and Atmospheric Administration, and NASA.Hence, option (a) is the correct answer.

Source: https://vajiramias.com/current-affairs/mosaic-mission/5d1ee9531d5def5a80bb3d5f/

Q43. With respect to the “Jal Jeevan Mission”, consider the following statements:

1. Its objective is to ensure Har Ghar Jal (piped water supply) to all rural households by 2024. 2. It is implemented by the Department of Drinking Water and Sanitation under the Jal Shakti Mantralaya.

Which of the statements given above is/are correct? a. 1 only b. 2 only c. Both 1 and 2 d. Neither 1 nor 2

Answer: c

Explanation:

Statement 1 is correct:

The mission objective is to ensure Har Ghar Jal (piped water supply) to all rural households by 2024 under the Jal Jeevan Mission.

Statement 2 is correct:

Its executing agency is the Department of Drinking Water and Sanitation under the Jal Shakti Mantralaya. This Mission will focus on integrated demand and supply-side management of water at the local level, including the creation of local infrastructure for source sustainability like rainwater harvesting, groundwater recharge and management of household wastewater for reuse in agriculture.The Mission will converge with other Central and State Government Schemes to achieve its objectives of sustainable water supply management across the country..

Source: https://vajiramias.com/current-affairs/jal-jeevan-mission/5d205c6b1d5def5a7ceb58c5/ Q44. With respect to “New Space India Limited (NSIL)”, consider the following statements:

1. It has been incorporated as a commercial arm of Department of Space. 2. It is the first commercial entity of the Department of Space (DoS) to tap the benefits of the Research & Development carried out by ISRO.

Which of the statements given above is/are correct? a. 1 only b. 2 only c. Both 1 and 2 d. Neither 1 nor 2

Answer: a

Explanation:

Statement 1 is correct:

New Space India Limited (NSIL) has been incorporated as a new commercial arm of Department of Space to tap the benefits of the Research & Development carried out by ISRO.The Company will spearhead commercialization of various space products including the production of launch vehicles, transfer to technologies and marketing of space products. Its headquarters is in Bengaluru. It has an authorized capital of ₹10 crores and a paid-up capital of ₹1 crore.

Statement 2 is incorrect:

NSIL is the second commercial entity of the Department of Space (DoS) after Antrix Corporation Limited, which was set up in 1992 to market the products and services of the Indian Space Research Organisation (ISRO).

Source: https://vajiramias.com/current-affairs/new-space-india-limited- nsil/5d2050831d5def5a84ea6551/

Q45. With respect to the “Pradhan Mantri Matsya Sampada Yojana (PMMSY), consider the following statements:

1. Its objective is to establish a robust fisheries management framework and check gaps in the value chain. 2. At present share of marine fish production in India’s total fish production is more than Inland.

Which of the statements given above is/are correct? a. 1 only b. 2 only c. Both 1 and 2 d. Neither 1 nor 2 Answer: a

Explanation:

Statement 1 is correct:

The government proposed a Pradhan Mantri Matsya Sampada Yojana (PMMSY) to establish a robust fisheries management framework and check gaps in the value chain.

Statement 2 is incorrect:

The total fish production during 2017-18 is estimated to be 12.60 million metric tonnes, of which nearly 65% is from inland sector and about 50% of the total production is from culture fisheries, and constitutes about 6.3% of the global fish production. Paradigm shifts in terms of increasing contributions from inland sector and further from aquaculture have been significant over the years.With high growth rates, the different facets, viz., marine fisheries, coastal aquaculture, inland fisheries, freshwater aquaculture, and coldwater fisheries are contributing to the food basket, health, economy, exports, employment and tourism of the country.

Source: https://vajiramias.com/current-affairs/pradhan-mantri-matsya-sampada-yojana- pmmsy/5d2052651d5def5a860ef0fe/

Q46. With reference to the UNESCO World Heritage Sites, consider the following statements:

1. India has a total of 17 world heritage sites.

2. Recently Jaipur listed as UNESCO World Heritage Site.

Which of the statements given above is/are correct? a. 1 only b. 2 only c. Both 1 and 2 d. Neither 1 nor 2

Answer: b

Explanation:

India gets its 38 th UNESCO World Heritage Site as Pink City Jaipur. India’s nomination of the Jaipur City, got inscribed on the World Heritage List of UNESCO during the 43rd Session of the World Heritage Committee held at Baku, Azerbaijan. The nomination of Jaipur City has successfully been done by complying with the various UNESCO guidelines of 2017. With Successful inscription of Jaipur City, India has 38 world heritage sites, that include 30 Cultural properties, 7 Natural properties and 1 mixed site. Jaipur was founded in 1727 CE under Sawai Jai Singh II. Hence only statement 2 is correct.

Value Addition The other sites now part of the UNESCO list are the Dilmun Burial Mounds in Bahrain, Budj Bim Cultural Landscape in Australia, Archaeological Ruins of Liangzhu City in China, Ombilin Coal Mining Heritage of Sawahlunto in Indonesia, Mozu-Furuichi Kofun Group in Japan and the Plain of Jars in Laos.

Source: https://vajiramias.com/current-affairs/unesco-world-heritage- tag/5d2174891d5def5a7f80447e/

Q47. With reference to the Earth Alliance, consider the following statements:

1. It is an initiative spearheaded by the UN Climate Change secretariat. 2. It will provide grants, educational opportunities and fund campaigns and films, as well as work with grassroots organizations and individuals in places most affected by biodiversity loss and climate change.

Which of the statements given above is/are correct? a. 1 only b. 2 only c. Both 1 and 2 d. Neither 1 nor 2

Answer: b

Explanation:

Statement 1 is incorrect:

Earth Alliance is a non-profit environmental organization. It will work globally to protect ecosystems and wildlife, ensure climate justice, support renewable energy and secure indigenous rights to the benefit of all life on Earth”.

Statement 2 is correct:

It will provide grants, educational opportunities and fund campaigns and films, as well as work with grassroots organizations and individuals in places most affected by biodiversity loss and climate change. Key people involved are Leonardo DiCaprio and billionaire investors and philanthropists Laurene Powell Jobs and Brian Sheth.

Value Addition

Momentum for Change is an initiative spearheaded by the UN Climate Change secretariat to shine a light on the enormous groundswell of activities underway across the globe that are moving the world toward a highly resilient, low-carbon future. Momentum for Change recognizes innovative and transformative solutions that address both climate change and wider economic, social and environmental challenges.

Source: https://vajiramias.com/current-affairs/earth-alliance/5d21787c1d5def5a7ec3ee35/

Q48. Recently in news, e-MASIHA is related to: a. It is an application developed by the Indian Haj mission in Saudi Arabia to store online the health status and medical history of visiting Haj pilgrims. b. Digitizing internal work-flows and processes of hospitals enabling efficient and online delivery of health care services across India. c. It is an internet-based survey records data directory of the state of Assam. d. A mobile application to support the professional communication and collaboration needs of the entire police department of the state of Telangana.

Answer: a

Explanation:

Indian Haj mission in Saudi Arabia has adopted digital technology to reach out to a large number of pilgrims as part of the ’s initiative of Digital India. Indian Haj Information System (Haj App) has been developed to receive feedback, a grievance from Hajis. The app also connects with the pilgrims to Haj service coordinators known as Qadimul Hajjaj of their respective states. Qadimul Hajjaj are government employees who are deputed to assist pilgrims during Haj season. Health status and medical history of visiting pilgrims are being stored at online “e-MASIHA” (Medical Assistance System for Indian Hajis Abroad). E-MASIHA would retrieve pilgrims’ health information to apply appropriate treatment whenever he or she visits Indian medical facilities during Haj. Hence, option (a) is the correct answer.

Source: https://vajiramias.com/current-affairs/haj/5d2180f01d5def5a81e8194e/

Q49. Researchers have recently developed “Deep-CEE”, an Artificial Intelligence-powered tool to identify: a. Deep ocean minerals b. Uranium deposits in the world c. Galaxy clusters quickly d. None of the above

Answer: c

Explanation:

Researchers have developed “Deep-CEE”, an Artificial Intelligence-powered tool to identify galaxy clusters quickly. Galaxy clusters are some of the most massive structures in the cosmos, but despite being millions of lightyears across, they can still be hard to spot. Researchers at Lancaster University have turned to artificial intelligence for assistance, developing Deep-CEE, a novel deep learning technique to speed up the process of finding them. Full name is Deep Learning for Galaxy Cluster Extraction and Evaluation (Deep-CEE). This Artificial Intelligence-powered tool that has been trained to “look” at colour images and identify galaxy clusters quickly. The “Deep-CEE” model is based on neural networks, which are designed to mimic the way a human brain learns to recognise objects by activating specific neurons when visualising distinctive patterns and colours. Hence, option (c) is the correct answer.

Source: https://vajiramias.com/current-affairs/deep-cee/5d2198d21d5def5a87c28f62/

Q50. Abujhmad forests often mentioned in news, is located in which of the following States? a. Kerala b. c. Nagaland d. Chhattisgarh

Answer: d

Explanation:

Nearly 7,000 Central Reserve Police Force (CRPF) personnel are to be deployed by the Union Home Ministry in the inaccessible areas of Abujhmad in south Chhattisgarh to fill the “security vacuum” against Maoists. The Abujhmad forests in south Chhattisgarh are not on the revenue map and the dense jungles are used by Maoists as their hideouts. Abujhmad forests is a hilly forest area in Chhattisgarh, covering Narayanpur district, Bijapur district and Dantewada district. Hence, option (d) is the correct answer.

Source: https://vajiramias.com/current-affairs/abujhmad-forests/5d21a0fd1d5def5a84ea85b2/

Q51. With reference to the short duration discussion in Rajya Sabha, consider the following statements:

1. The Chairman decides the admissibility of the notice. 2. There is no voting at the end of the discussion but the member who raises the discussion has a right of reply.

Which of the statements given above is/are correct? a. 1 only b. 2 only c. Both 1 and 2 d. Neither 1 nor 2

Answer: a

Explanation:

The short duration discussion is a procedural device to enable members to raise discussion on matters of urgent public importance without a formal motion on a matter of urgent public importance. Notice to raise a short duration discussion is to be given in writing, addressed to the Secretary-General, specifying clearly and precisely the matter to be raised. The notice is required to be accompanied by an explanatory note stating reasons for raising the discussion and supported by signatures of at least two other members. The Chairman decides the admissibility of the notice. The Chairman may refuse to admit the notice. Hence statement 1 is correct.

The Chairman may allow such time not exceeding two and a half hours for holding such discussion.Procedure for raising the discussion are the first member or in his absence the second one and so on, in whose name the discussion stands, makes a short statement.Thereafter, members are called to speak party-wise. At the end of the discussion, the Minister concerned replies and the discussion is concluded.There is no formal motion before the House nor is there any voting. The member who raises the discussion has no right of reply. Hence statement 2 is incorrect.

Source: https://vajiramias.com/current-affairs/short-duration- discussion/5d230c031d5def5a7ceb9db3/

Q52. With respect to the International Atomic Energy Agency (IAEA), consider the following statements:

1. It is a specialized agency of the United Nations Organization which works for the safe, secure and peaceful uses of nuclear science and technology. 2. The IAEA and its former Director-General, Mohamed El Baradei, were jointly awarded the Nobel Peace Prize in 2005.

Which of the statements given above is/are correct? a. 1 only b. 2 only c. Both 1 and 2 d. Neither 1 nor 2

Answer: b

Explanation:

Statement 1 is incorrect:

International Atomic Energy Agency is widely known as the world’s “Atoms for Peace and Development” organization within the United Nations family. It is the world’s central intergovernmental forum for scientific and technical cooperation in the nuclear field. It works for the safe, secure and peaceful uses of nuclear science and technology.Though established as an autonomous organisation, independently of the United Nations through its own international treaty, the IAEA Statute, the IAEA reports to both the United Nations General Assembly and Security Council. Its Headquarters is in Vienna, Austria. The IAEA has 171 member states. Most UN members and the Holy See are Member States of the IAEA.

Statement 2 is correct:

The IAEA and its former Director-General, Mohamed ElBaradei, were jointly awarded the Nobel Peace Prize in 2005. Value Addition

• The 15 Specialized agencies of the UN are: • Food and Agriculture Organization (FAO) • International Civil Aviation Organization (ICAO) • International Fund for Agricultural Development (IFAD) • International Labour Organization (ILO) • International Maritime Organization (IMO) • International Monetary Fund (IMF) • International Telecommunication Union (ITU) • United Nations Educational, Scientific and Cultural Organization (UNESCO) • United Nations Industrial Development Organization (UNIDO) • Universal Postal Union (UPU) • World Bank Group (WBG) • World Health Organization (WHO) • World Intellectual Property Organization (WIPO) • World Meteorological Organization (WMO) • World Tourism Organization (UNWTO)

Source: https://vajiramias.com/current-affairs/international-atomic-energy-agency- iaea/5d230eba1d5def5a7ceb9dfc/

Q53. With respect to “Vande Bharat Express”, consider the following statements:

1. It is the first locomotive-less train in the country. 2. It is India’s first ultra-high-speed train which will run from Delhi to Varanasi.

Which of the statements given above is/are correct? a. 1 only b. 2 only c. Both 1 and 2 d. Neither 1 nor 2

Answer: a

Explanation:

Statement 2 is incorrect:

Vande Bharat Express, earlier named as Train 18, is India’s fastest indigenous train which has been built by the Integral Coach Factory, Chennai under the Government’s Make in India initiative. It is India’s first semi-high-speed train which will run from Delhi to Varanasi, with halts at Kanpur and Allahabad, at a maximum speed of 160 kmph by covering the distance in 8 hours.It is equipped with world-class passenger amenities like On-board wifi entertainment, GPS based passenger information system, CCTVs, bio-vacuum toilets, rotating chairs in executive class, etc. Statement 1 is correct:

It is also the first locomotive-less train in the country.

Source:https://vajiramias.com/current-affairs/vande-bharat-express/5d230da21d5def5a7ceb9de8/

Q54. Prakash Purab is one of the most sacred festivals in Sikhism and celebrates the birth of who among the following Sikh Gurus? a. Guru Tegh Bahadur b. Guru Arjan c. Guru Nanak d. Guru Angad

Answer: c

Explanation:

Prakash Purab is also known as Guru Nanak Gurpurab, Guru Nanak’s Prakash Utsav and Guru Nanak Jayanti. This is one of the most sacred festivals in Sikhism and celebrates the birth of the first Sikh Guru, Guru Nanak Dev. The Sikhs have been celebrating Guru Nanak’s Gurpurab on the Full Moon Day (Pooranmashi or Purnima) of the Lunar Month Kartik which falls around November. Hence option (d) is correct.

Source: https://vajiramias.com/current-affairs/parkash-purab/5d230b161d5def5a860f3692/

Q55. Which of the following pairs is/are correctly matched?

Tribe State

1. Kondh Assam

2. Bhils Kerala

3. Khasi Meghalaya

Select the correct answer using the code given below: a. 3 only b. 1 and 2 only c. 2 and 3 only d. 1, 2 and 3

Answer: a Explanation:

Pair (1) is not correctly matched: The Kondhs constitute one of the major tribes of Odisha. The word Kondh means ‘mountaineer’ derived from the Telugu word ko or ku, signifying a hill or mountain. These tribals still continue to call themselves as ‘Kui people’ and their hills as ‘kui country’. The kondhs are a Dravidian tribe. Traditionally hunter-gatherers, they are divided into Desia Kondhs, Dongria Kondhs and Kutia Kondhs.

Pair (2) is not correctly matched: Bhils or Bheels are an Indo-Aryan speaking ethnic group in West India. They speak the Bhil languages, a subgroup of the Western Zone of the Indo-Aryan languages. This tribal community in India is mostly spotted in the Aravali Ranges of Sirohi in Udaipur and some places of Dungarpur and Banswara districts of Rajasthan.

Pair (3) is correctly matched: The Khasi people are an indigenous ethnic group of Meghalaya in north- eastern India, with a significant population in the bordering state of Assam, and in certain parts of Bangladesh.

Source: https://vajiramias.com/current-affairs/kui-tribals/5d230abc1d5def5a7ceb9da0/

Q56. Which of the following is the correct ascending sequence of trophic levels in a food chain? a. Desert Locust - Frog- Snake- Hawk b. Frog - Desert Locust - Snake- Hawk c. Desert Locust - Frog – Hawk - Snake d. Desert Locust - Hawk - Frog- Snake

Answer: a

Explanation:

Locusts are part of a large group of insects commonly called grasshoppers which have big hind legs for jumping. Locusts belong to the family called Acrididae. Locusts differ from grasshoppers in that they have the ability to change their behaviour and habits and can migrate over large distances. The Desert Locust is one of about a dozen species of short-horned grasshoppers (Acridoidea) that are known to change their behaviour and form swarms of adults or bands of hoppers (wingless nymphs). The swarms that form can be dense and highly mobile. The Latin name for Desert Locust is Schistocerca gregaria (Forskal).

Grasshoppers (Caelifera) are members of the class Insecta and classified under the order Orthoptera, which also contains locusts, katydids and crickets. There are more than 11,000 known species of grasshoppers inhabiting every continent on Earth except Antarctica.A food chain is a linear sequence of organisms through which nutrients and energy pass as one organism eats another. In a food chain, each organism occupies a different trophic level, defined by how many energy transfers separate it from the basic input of the chain.Hence, option (a) is the correct answer.

Source: https://vajiramias.com/current-affairs/desert-locust/5d25afff1d5def6087ed6926/

Q57. With reference to the Law for Refugees in India, consider the following statements:

1. India is a signatory to the 1951 UN Convention on the Status of Refugees but no to the 1967 Protocol. 2. There is no national law on Refugee at present in India.

Which of the statements given above is/are correct? a. 1 only b. 2 only c. Both 1 and 2 d. Neither 1 nor 2

Answer: b

Explanation:

India is not a signatory to the 1951 United Nations Convention on the Status of Refugees and the 1967 Protocol thereon. There is no national law on Refugee at present. Only Standard Operating Procedure are issued by Ministry of Home Affairs to deal with foreign nationals in India, who claim to be refugees. The Standard Operating Procedure issued in 2011 stipulates that a foreigner to whom Long Term Visa (LTV) is permitted by the Ministry of Home Affairs will be allowed to take up any employment in the private sector or to undertake studies in any academic institution. Hence only statement 2 is correct.

Source: https://vajiramias.com/current-affairs/rohingyas/5d25b0e81d5def608c3778da/

Q58. With respect to Pradhan Mantri Gram Sadak Yojana (PMGSY-III), consider the following statements:

1. It involves a consolidation of through routes and major rural links connecting habitations to Gramin Agricultural markets (GrAMs). 2. The funds would be shared in the ratio of 60:40 between the Centre and State for all States except for 8 North Eastern and 3 Himalayan States for which it is 90:10. Which of the statements given above is/are correct? a. 1 only b. 2 only c. Both 1 and 2 d. Neither 1 nor 2

Answer: c

Explanation:

The Cabinet Committee on Economic Affairs has recently approved the launch of the third phase of the Pradhan Mantri Gram Sadak Yojana (PMGSY-III). Under the PMGSY-III Scheme, it is proposed to consolidate 1,25,000 Km road length in the States.

Statement 1 is correct:

PMGSY-III involves a consolidation of through routes and major rural links connecting habitations to Gramin Agricultural markets (GrAMs), higher secondary schools and hospitals.

Statement 2 is correct:

It will entail an estimated cost of Rs 80,250 crore. The funds would be shared in the ratio of 60:40 between the Centre and State for all States except for 8 North Eastern and 3 Himalayan States for which it is 90:10. Its implementation period is 2019-20 to 2024-25.

Source: https://vajiramias.com/current-affairs/pmgsy-phase-iii/5d26c7111d5def6089b90ebd/

Q59. Kharchi Puja is celebrated in the temple premises of fourteen gods and it is one of the most popular festivals in which of the following States? a. b. Andhra Pradesh c. Bihar d.

Answer: d

Explanation:

Kharchi Puja is one of the most popular festivals in Tripura. It is celebrated at Agartala (Puran Agartala) in the temple premises of fourteen gods. It’s a week-long royal Puja which falls in the month of July on the eighth day of the new moon.The word Kharchi is derived from the word Khya which means earth. Kharchi Puja is basically done to clean the postmenstrual phase of mother earth’s menstruation. On the day of the Puja, the fourteen gods are carried to river Saidra by the members of chantai. The gods are bathed in the holy water and are brought back to the temple. Animal sacrifice is also an important part of this festival.Hence option (d) is correct. Source: https://vajiramias.com/current-affairs/kharchi-puja/5d26c4df1d5def608c379cff/

Q60. With reference to the Microsoft HoloLens, consider the following statements:

1. It is known under development as Project Baraboo. 2. It is a mixed reality holographic headset.

Which of the statements given above is/are correct? a. 1 only b. 2 only c. Both 1 and 2 d. Neither 1 nor 2

Answer: c

Explanation:

Microsoft HoloLens, known under development as Project Baraboo, is a stand-alone mixed reality holographic headset specifically designed to be used on the Windows 10 platform. HoloLens allows users to experience 3D holographic images as though they are a part of their environment. This level of immersion enables new forms of computing in which the user’s desktop could be the living room. Along with its CPU and GPU the unit features a new first: a holographic processing unit (HPU) that is responsible for the processing that integrates real world and holographic data. Hence both statements are correct.

Source: https://vajiramias.com/current-affairs/hololens/5d26c7ed1d5def60850a56a3/

Immersive Experience

• An immersive experience is a perception of being in one place when you are actually in another. It is essentially the suspension of reality, even if just for a few moments. • The three pillars of immersive experiences are visual quality, sound quality, and intuitive interactions. Full immersion can only be achieved by simultaneously focusing on these pillars. • There are a number of different technologies and hardware that enable immersive experiences, e.g. Microsoft HoloLens.

Q61. With reference to the Witness Protection Scheme, 2018, consider the following statements:

1. It provides for the protection of witnesses based on the threat assessment. 2. It provides for a State Witness Protection Fund for meeting the expenses of the scheme. Which of the statements given above is/are correct? a. 1 only b. 2 only c. Both 1 and 2 d. Neither 1 nor 2

Answer: c

Explanation:

Statement 1 is correct:

Witness Protection Scheme, 2018 provides for protection of witnesses based on the threat assessment. Protection measures inter alia include: Protection/change of identity of witnesses, their relocation, installation of security devices at the residence of witnesses, usage of specially designed Court rooms, etc. Three categories of witness as per threat perception:

• Category A: Where the threat extends to life of witness or his family members, during investigation/trial or thereafter. • Category B: Where the threat extends to safety, reputation or property of the witness or his family members, during the investigation/trial or thereafter. • Category C: Where the threat is moderate and extends to harassment or intimidation of the witness or his family member’s, reputation or property, during the investigation/trial or thereafter.

Statement 2 is correct:

The Scheme provides for a State Witness Protection Fund for meeting the expenses of the scheme. This fund shall be operated by the Department/Ministry of Home under State/UT Government.

Source: https://vajiramias.com/current-affairs/witness-protection- scheme/5d26c9041d5def6087ed8689/

Q62. In which of the following companies where the government has at least a 51% stake?

1. Bharat Heavy Electricals 2. Engineers India Ltd 3. Oil and Natural Gas Corporation

Select the correct answer using the code given below: a. 1 only b. 1 and 2 only c. 2 and 3 only d. 1, 2 and 3 Answer: d

Explanation:

At present there are more than two dozen CPSEs that are widely held by the public with a government stake of less than or close to 60%. These include 'maharatna' and 'navratna' CPSEs like Engineers India Ltd (EIL-52%), Indian Oil Corporation (IOC-52.18%), Bharat Petroleum Corporation (BPCL-53.29%), Gail India (52.64%), Oil and Natural Gas Corporation (ONGC-64.25%), Power Finance Corporation (PFC- 59.05%), Powergrid Corporation (PGCIL-55.37%), NTPC, Shipping Corporation of India (SCI-63.75%), Bharat Heavy Electricals (BHEL-63.17%), NBCC (68.18%), Container Corporation (Concor 54.80%). Recently the government approved the strategic disinvestment of Bharat Petroleum Corp Ltd (BPCL) and Shipping Corporation of India. Also on sale will be a 31% stake in Container Corporation of India (Concor) along with management control.Hence, option (d) is the correct answer.

Source: https://vajiramias.com/current-affairs/pawan-hans/5d28595d1d5def60843ce3af/

Q63. With respect to “Mission Raksha Gyan Shakti”, recently seen in the news, consider the following statements:

1. It aims to inculcate Intellectual Property Right (IPR) culture in Indian defence manufacturing ecosystem. 2. The Directorate General of Quality Assurance (DGQA), Ministry of Defence is the implementing agency.

Which of the statement/s given above statements is/are correct? a. 1 only b. 2 only c. Both 1 and 2 d. Neither 1 nor 2

Answer: c

Explanation:

The Department of Defence Production has recently instituted a new framework titled ‘Mission Raksha Gyan Shakti’ which aims to inculcate Intellectual Property Right (IPR) culture in Indian defence manufacturing ecosystem. The Directorate General of Quality Assurance (DGQA) has been entrusted with the responsibility of coordinating and implementing the programme. Hence both the statements are correct.

The Action Plan 2019-20 of MRGS has also been approved by Raksha Mantri Shri Rajnath Singh, which includes training of additional 20,000 personnel from Public / Private Sector Industry and Armed Forces on relevance of IP Culture to Indian Defence manufacturing industry and instituting a framework which enables full utilization of new ideas and innovation towards achieving self-reliance in defence sector.

Source: https://vajiramias.com/current-affairs/mission-raksha-gyan-shakti- mrgs/5d285a881d5def608ecd0792/ Q64. Which of the following countries are members of the Commonwealth of British Nations?

1. India 2. Maldives 3. Afghanistan 4. Rwanda 5. Bangladesh

Select the correct answer using the code given below: a. 1 and 5 only b. 1, 2 and 3 only c. 2, 3 and 4 only d. 1, 4 and 5 only

Answer: d

Explanation:

The Commonwealth is a voluntary association of 53 independent and equal sovereign states, nearly all of them former territories of the British Empire. Headquarters is in London, United Kingdom. Member countries: 53 countries are members of the Commonwealth. The last country to join the Commonwealth was Rwanda in 2009. All members have an equal say – regardless of size or economic stature. This ensures even the smallest member countries have a voice in shaping the Commonwealth. Commonwealth Heads of Government Meeting (CHOGM): Every two years, members meet to discuss issues affecting the Commonwealth and the wider world at the Commonwealth Heads of Government Meeting (CHOGM). Hence, option (d) is the correct answer.

Source: https://vajiramias.com/current-affairs/commonwealth-and- maldives/5d28617e1d5def6086fb3e9b/

Q65. With reference to the amendments to the Protection of Children from Sexual Offences Act 2012, (POCSO), consider the following statements:

1. It makes punishment more stringent for committing sexual crimes against children including the death penalty. 2. It defines a child as any person below 16 years of age.

Which of the statements given above is/are correct? a. 1 only b. 2 only c. Both 1 and 2 d. Neither 1 nor 2

Answer: a

Explanation:

Statement 1 is correct:

Cabinet has approved the Amendments to the Protection of Children from Sexual Offences, POCSO, Act, 2012 to discourage the trend of child sexual abuse by acting as a deterrent due to strong penal provisions incorporated in the Act. It will make punishment more stringent for committing sexual crimes against children including the death penalty. The amendments also provide for levy of fines and imprisonment to curb child pornography. The new definition of child pornography reads, “Any visual depiction of sexually explicit conduct involving a child which include photographs, video, digital or computer-generated image indistinguishable from an actual child and an image created, adapted or modified but appear to depict a child”. Neither Section 67 of the IT Act nor Section 293 of the Indian Penal Code define child pornography. The POCSO Act, 2012 was enacted to Protect the Children from Offences of Sexual Assault, Sexual harassment and pornography with due regard for safeguarding the interest and well-being of children.

Statement 2 is incorrect:

The Act defines a child as any person below eighteen years of age, and regards the best interests and welfare of the child as a matter of paramount importance at every stage, to ensure the healthy physical, emotional, intellectual and social development of the child. The act is gender-neutral.

Source: https://vajiramias.com/current-affairs/amendments-to-pocso- act/5d2861d91d5def608ecd080a/ https://vajiramias.com/current-affairs/special-courts-for-pocso-cases/5d3aa6731d5def7124b0839e/

Q66. With respect to “Operation Thirst”, consider the following statements:

1. It is an All India Drive to crackdown selling of unauthorised Packaged Drinking Water in Railway premises. 2. It is launched by the Ministry of Jal Shakti.

Which of the statements given above is/are correct? a. 1 only b. 2 only c. Both 1 and 2 d. Neither 1 nor 2

Answer: a

Explanation:

Statement 2 is incorrect: Railway Protection Force (RPF) launched “Operation Thirst”, an All India Drive to crackdown selling of unauthorised Packaged Drinking Water.

Statement 1 is correct:It aims to curb the menace of unauthorized Packaged Drinking Water (PDW) in Railway premises, an all India drive named “Operation Thirst”. During this, all the Zonal Principal Chief Security Commissioners (PCSCs) were asked to crackdown these unauthorised activities. Almost all major stations over Indian Railway were covered during this operation.

Source:https://vajiramias.com/current-affairs/operation-thirst/5d2831e01d5def60817d6010/

Q67. With respect to “Abu Musa”, consider the following statements:

1. It is an island in the Gulf of Oman. 2. It is near the entrance to the Strait of Hormuz.

Which of the statements given above is/are correct? a. 1 only b. 2 only c. Both 1 and 2 d. Neither 1 nor 2

Answer: b

Explanation:

Statement 1 is incorrect: Abu Musa is an island in the eastern Persian Gulf.

Statement 2 is correct: It is near the entrance to the Strait of Hormuz.

The island is administered by Iran as part of its province of Hormozgan, but is also claimed by the United Arab Emirates as a territory of the emirate of Sharjah.

Source: https://vajiramias.com/current-affairs/abu-musa/5d2836fd1d5def608ecd03cb/

Q68. With reference to the Second India-Russia Strategic Economic Dialogue (IRSED), consider the following statements:

1. The IRSED was established during the 2018 BRICS summit in Johannesburg. 2. The First India-Russia Strategic Economic Dialogue was held in Moscow

Which of the statements given above is/are correct? a. 1 only b. 2 only c. Both 1 and 2 d. Neither 1 nor 2

Answer: d

Explanation:

The Second India-Russia Strategic Economic Dialogue (IRSED) was recently held in New Delhi. The meeting focussed on 6 core areas covering Transport, Agriculture, Small and Medium Business, Digital Technologies, Finance, Tourism & Connectivity. The IRSED was established following a bilateral MoU signed between NITI Aayog and the Ministry of Economic Development of Russia during the 19th edition of the Annual India-Russia Bilateral Summit held in 2018 in New Delhi. The First India-Russia Strategic Economic Dialogue was held in St. Petersburg in November 2018. Hence both statements are incorrect.

Source: https://vajiramias.com/current-affairs/second-india-russia-strategic-economic- dialogue/5d29964c1d5def6089b97084/

Q69. The Yellow Sea is located between which of the following countries? a. China and Russia b. Vietnam and the Philippines c. China and Japan d. None of the above

Answer: d

Explanation:

The Yellow Sea is located between China and Korea. The name is given to the northern part of the East China Sea, which is a marginal sea of the Pacific Ocean. It is located between mainland China and the Korean Peninsula. The rivers that flow into the Yellow Sea carry so much mineral-rich soil that the water actually turns yellow. The Yellow Sea Marine Ecoregion includes the Yellow Sea, Bohai Sea and part of the East China Sea. It is one of the world's largest areas of continental shelf covered in shallow water, providing for rich fishing grounds and an important site for migratory birds. But decades of pollution is turning the sea to a colour other than yellow. Hence, option (d) is the correct answer.

Source:https://vajiramias.com/current-affairs/kim-jong-un/5d2999bc1d5def60817d8ee4/

Q70. Which is of the following best describes “301 Investigation” by the United States of America (USA)? a. It is a probe employed as a precursor to tariffs and other trade measures against a country. b. It is an annual report to the US Congress on International Religious Freedom which describes the status of religious freedom. c. It is the U.S. inquiry on the Iranian nuclear enrichment program. d. None of the above

Answer: a

Explanation:

“301 investigation” is a probe employed as a precursor to tariffs and other trade measures against a country. A Special 301 Report is prepared annually by the USTR and is different from a 301 investigation. Section 301 of the U.S. Trade Act (1974) was also used to authorise a 2017 probe that resulted in tariffs on Chinese exports to the U.S. from July 2018. Section 301 of the U.S. Trade Act of 1974 authorizes the President to take all appropriate action to obtain the removal of any act or practice of a foreign government that burdens or restricts U.S. commerce. Section 301 cases can be self-initiated by the United States Trade Representative (USTR) or as the result of a petition filed by a firm or industry group. Hence, option (a) is the correct answer. Source: https://vajiramias.com/current-affairs/301-probe-united- states/5d297ce01d5def60843d0b50/

Q71. India-based Neutrino Observatory (INO) is located at which of the following places? a. Ladakh b. Theni c. Wayanad d. Nagpur

Answer: b

Explanation:

The Government of India has approved a project to build the India-based Neutrino Observatory (INO) at Pottipuram in the Theni District of Tamil Nadu. The project aims to set up a 51000 ton Iron Calorimeter (ICAL) detector to observe naturally occurring atmospheric neutrinos in a cavern at the end of an approximately 2 km long tunnel in a mountain. This will help to reduce the noise from cosmic rays that is ever-present over-ground and which would outnumber the rare neutrino interactions even in a detector as large as ICAL. According to India, The INO project does not disturb the ecosystem around the site and does not release any radiation, as it does not have any radioactive substance. It measures cosmic rays. There is no other neutrino detector anywhere in India at present. ICAL at INO would be the first of its type. Hence, option (b) is the correct answer.

Source: https://vajiramias.com/current-affairs/india-based-neutrino-observatory- ino/5d298ea31d5def6086fb6756/

Q72. With reference to the “2019 Global Multidimensional Poverty Index”, consider the following statements:

1. It was released by the UN Development Programme, the Oxford Poverty and Human Development Initiative. 2. Of the 10 selected countries in the Index, India and Cambodia are the countries that reduced their Multidimensional Poverty Index (MPI) values the fastest.

Which of the statements given above is/are correct? a. 1 only b. 2 only c. Both 1 and 2 d. Neither 1 nor 2

Answer: c

Explanation: According to the report titled “2019 global Multidimensional Poverty Index” released by UN, India lifted 271 million people out of poverty between 2006 and 2016. The 2019 global Multidimensional Poverty Index was released by the UN Development Programme, the Oxford Poverty and Human Development Initiative. In the 101 countries studied – 31 low income, 68 middle income and 2 high income –, 1.3 billion people are multidimensionally poor.

The report identifies 10 countries, with a combined population of around 2 billion people that have shown statistically significant progress towards achieving Sustainable Development Goal 1, namely ending poverty in all its forms, everywhere. Of the 10 selected countries, India and Cambodia reduced their MPI values the fastest. India’s MPI value reduced from 0.283 in 2005-06 to 0.123 in 2015-16. Hence both statements are correct.

Source: https://vajiramias.com/current-affairs/2019-global-multidimensional-poverty- index/5d29763f1d5def6086fb64e7/

Q73. With reference to the Rewa Ultra Mega Solar, consider the following statements:

1. It is Asia’s largest solar power project located in Rewa, Madhya Pradesh. 2. It is the first project to get funding from Clean Technology Fund (CTF) in India.

Which of the statements given above is/are correct? a. 1 only b. 2 only c. Both 1 and 2 d. Neither 1 nor 2

Answer: c

Explanation:

Rewa Ultra Mega Solar is a solar park in the Gurh tehsil of Rewa district of Madhya Pradesh. The project was commissioned with 750 MW capacity in July 2018. Rewa Ultra Mega Solar Limited (RUMSL), the implementing agency of the project, is a joint venture between the Madhya Pradesh Urja Vikash Nigam Limited (MPUVNL) and the Solar Energy Corporation of India (SECI). It is the first project to get funding from Clean Technology Fund (CTF) in India. It is also the first solar park in India to get World Bank funding. Hence both statements are correct.

Source: https://vajiramias.com/current-affairs/rewa-ultra-mega-solar/5d2973e31d5def6087edde8b/

Ultra Mega Solar Power Projects Ultra Mega Solar Power Projects, also known as Ultra Mega Solar Parks, are a series of solar power projects planned by the Ministry of New and Renewable Energy of the Union Government of India. Each power project has a minimum capacity of 500 MW. In 2014, the Government of India introduced a scheme to establish at least 25 solar parks and Ultra Mega Solar Power Projects, adding over 20 GW of installed solar power capacity. In 2017, the Union Cabinet increased the total number of planned solar parks to 50 with a total capacity of 40 GW. The Central Government provides financial support for the construction of these solar projects. Q74. With reference to the overseas bonds, consider the following statements:

1. A government bond is a form of debt that the government undertakes wherein it issues bonds with the promise to pay periodic interest payments and repay the entire face value of the bond on the maturity date. 2. Government of India issues bonds in the domestic as well as international market.

Which of the statements given above is/are correct? a. 1 only b. 2 only c. Both 1 and 2 d. Neither 1 nor 2

Answer: a

Explanation:

Statement 1 is correct:

A government bond or sovereign bond is a form of debt that the government undertakes wherein it issues bonds with the promise to pay periodic interest payments and also repay the entire face value of the bond on the maturity date.

Statement 2 is incorrect:

So far, the government of India has only issued bonds in the domestic market. Union Finance Minister announced in the Budget that the government plans to raise a portion of its gross borrowing from overseas markets. The government and RBI will reportedly finalise the plans for the overseas issue of sovereign bonds by September. The overseas borrowing programme allows the government to maintain its gradual reduction of the fiscal deficit. Borrowing overseas allows the government to raise funds in such a way that there is enough domestic credit available for the private sector. This would also have a beneficial impact on the demand for government securities in India.

Source: https://vajiramias.com/current-affairs/overseas-bonds/5d2aceb31d5def60817db6d9/

Q75. Which of the following pairs is/are correctly matched?

People Countries

1. Catalans Canada

2. Uyghurs China

3. Kurds Turkey

4. Rohingya Nepal

Select the correct answer using the code given below: a. 1 and 4 only b. 2 and 3 only c. 2, 3 and 4 only d. 1, 2 and 3 only

Answer: b

Explanation:

• The Uyghurs are a Turkic ethnic group who live in East and Central Asia. They live primarily in the Xinjiang Uyghur Autonomous Region of China, where they are one of 55 officially recognized ethnic minorities. • Catalans are an ethnic group native to Catalonia, a region spanning Spain. • The Kurds are one of the indigenous peoples of the Mesopotamian plains and the highlands in what is now south-eastern Turkey, northeastern Syria, northern Iraq, north-western Iran and south-western Armenia. • Rohingya are an ethnic group, largely comprising Muslims, who predominantly live in the Western Myanmar province of Rakhine.

Hence, option (b) is the correct answer.

Source: https://vajiramias.com/current-affairs/rohingya/5d2ac2c71d5def6080179517/

Q76. Consider the following statements with reference to the Spektr-RG:

1. It is a space observatory intended to observe black holes, neutron stars and magnetic fields. 2. It has been developed by the National Aeronautics and Space Administration (NASA).

Which of the statements given above is/are correct? a. 1 only b. 2 only c. Both 1 and 2 d. Neither 1 nor 2

Answer: a

Explanation:

Russia launched Spektr-RG, a space telescope from the cosmodrome in Baikonur, Kazakhstan, in a joint project with Germany. The Spektr-RG is a space observatory intended to observe black holes, neutron stars and magnetic fields. It has been developed by Russia with Germany. It was launched on a Proton-M rocket from the cosmodrome in Baikonur, Kazakhstan. The Spektr-RG intends to replace the Spektr-R, known as the “Russian Hubble”, which Roskosmos lost control of in January. Spektr-R was launched in 2011 to observe black holes, neutron stars and magnetic fields. Hence only statement 1 is correct. Source: https://vajiramias.com/current-affairs/spektr-rg/5d2ad57c1d5def60850adb52/

Q77. What is Elektorornis chenguangi, recently seen in the news? a. An extinct bird with an extra-long toe. b. An exoplanet c. Ghost particle' from space d. The only interstellar object detected passing through the Solar System.

Answer: a

Explanation:

Elektorornis chenguangi – an extinct bird with an extra-long toe – has been described in a study published in the journal Current Biology. In 2014 the fossil of Elektorornis chenguangi – a small bird which lived 99 million years ago, with a weird elongated toe – was discovered in Myanmar’s Hukawng Valley. The preserved toe measures less than half an inch from knuckle to claw-tip, making it 41% longer than the next longest digit on the creature’s foot. Its elongated toe structure has never been observed in other birds, living or extinct. Elektorornis chenguangi may have used the long, sensitive digit to probe cracks in trees for insects and grubs. Hence, option (a) is the correct answer.

Source: https://vajiramias.com/current-affairs/elektorornis- chenguangi/5d2add781d5def60850adc22/

Q78. With reference to the National Green Tribunal (NGT), consider the following statements:

1. The NGT Act, 2010 provides for a Chairperson and a minimum of 10 Expert Members and the equal number of Judicial Members. 2. Kolkata is the Principal Place of Sitting of the Tribunal.

Which of the statements given above is/are correct? a. 1 only b. 2 only c. Both 1 and 2 d. Neither 1 nor 2

Answer: d

Explanation:

Statement 1 is incorrect:

NGT was established under the National Green Tribunal Act 2010. Consequent to the enforcement of the National Green Tribunal Act, 2010, the National Environment Tribunal Act, 1995 and the National Environment Appellate Authority Act, 1997 have been repealed. It is a specialized body equipped with the necessary expertise to handle environmental disputes involving multi-disciplinary issues. The NGT Act, 2010 provides for a Chairperson and a minimum of 10 Expert Members (maximum 20) and also minimum 10 Judicial Members (maximum 20).

However, as long as each bench constituted in the tribunal consists of at least one Expert Member and one Judicial Member, the total number of Expert and Judicial Members in the overall NGT may be unequal (within the limit of minimum 10 and maximum 20).

Statement 2 is incorrect:

New Delhi is the Principal Place of Sitting of the Tribunal. Bhopal, Pune, Kolkata and Chennai shall be the other four place of sitting of the Tribunal. The Tribunal has the same powers as are vested in a civil court under the Code of Civil Procedure, 1908. The Tribunal shall not be bound by the procedure laid down under the Code of Civil Procedure, 1908, but shall be guided by principles of natural justice. The Tribunal is mandated to make and endeavour for disposal of applications or appeals finally within 6 months of the filing of the same. The Tribunal has the original jurisdiction over all civil cases where a substantial question relating to the environment, including enforcement of any legal right relating to environment is involved. An appeal against any order of the Tribunal shall lie to the Supreme Court.

Source: https://vajiramias.com/current-affairs/national-green-tribunal- ngt/5d2af70f1d5def6086fb9611/

Q79. In which of the following states is Vikramshila Gangetic Dolphin Sanctuary (VGDS) located? a. Uttar Pradesh b. West Bengal c. Bihar d. Madhya Pradesh

Answer: c

Explanation:

The Vikramshila Gangetic Dolphin Sanctuary (VGDS), from Sultanganj to Kahalganj on the Ganga in Bihar is the only dolphin sanctuary in the country. The population of the Ganges dolphin in VGDS from 150 to 200. National Waterway-1 connecting Haldia to Varanasi passes through the Vikramshila Gangetic Dolphin Sanctuary (VGDS). Hence, option (c) is the correct answer.

Source: https://vajiramias.com/current-affairs/vikramshila-gangetic-dolphin-sanctuary- vgds/5d2c1c031d5def608017c04a/

Q80. With reference to the Gurdwara Darbar Sahib at Kartarpur, consider the following statements:

1. It stands on the banks of the River Chenab. 2. It is the final resting place of Guru Nanak.

Which of the statements given above is/are correct? a. 1 only b. 2 only c. Both 1 and 2 d. Neither 1 nor 2

Answer: b

Explanation:

Gurdwara Darbar Sahib is located at Kartarpur in Pakistan’s Narowal district. It stands on the bank of the River Ravi, about 120 km northeast of Lahore. The shrine is visible from the Indian side. It was here that the First Sikh Guru (Nanak Dev) assembled a Sikh community and lived for 18 years until his death in 1539. It is the final resting place of Guru Nanak. The gurdwara was opened to pilgrims after Prime Minister Atal Bihari Vajpayee paid a visit to Pakistan in 1999. Since then Sikh Pilgrims are visiting the shrine regularly. Hence only statement 2 is correct.

Source: https://vajiramias.com/current-affairs/kartarpur-sahib/5d2c220c1d5def6087ee2dde/

Q81. The terms ‘Agent Smith and GravityRAT’, sometimes mentioned in the news recently are related to: a. Exoplanets b. Cryptocurrency c. Communication satellites d. Malware

Answer: d

Explanation:

According to Indian cybercrime officials, Indians constitute the highest number of victims of the recently detected Agent Smith malware. Agent Smith is embedded in apps available on Google Playstore, mostly connected to gaming, image editing or adult entertainment. It is believed to have infected over 25 million devices so far. Around 59% of those affected by Agent Smith are Indians.Maharashtra Cybercrime department has recently reported the malware “GravityRAT”. It was first detected by Indian Computer Emergency Response Team (CERT-In) in 2017. Hence, option (d) is the correct answer.

Value Addition

Ransomware is a kind of malware (software that damages the functions or gains unauthorized access to a computer system). It is used to encrypt important documents or files within a system (Crypto ransomware) or simply lock the original user out of the system (Locker ransomware). The user is then asked for a ransom in return for decrypting the files. Once the ransom is paid within a stipulated period, then the system is either unlocked or the system’s contents are deleted or the system is entirely corrupted. Unlike other cyber-attacks, in this form of attack, the user is notified of the attack. Ransomware spreads easily when it encounters unpatched or outdated software.

Source: https://vajiramias.com/current-affairs/agent-smith/5d3eb45f1d5def7126ede493/ Q82. With respect to “Seva Bhoj Yojna”, consider the following statements:

1. Financial Assistance under the scheme shall be given only to those institutions which are not in receipt of any Financial Assistance from the Central/State Government for the purpose of distributing free food. 2. It is a Central Sponsored Scheme by Ministry of Culture.

Which of the statements given above is/are correct? a. 1 only b. 2 only c. Both 1 and 2 d. Neither 1 nor 2

Answer: a

Statement 2 is incorrect:

It is a Central Sector Scheme. The Government of India is running Seva Bhoj Yojana for providing assistance to charitable religious institutions for serving free food to the public.

Statement 1 is correct:

Financial Assistance under the scheme shall be given only to those institutions which are not in receipt of any Financial Assistance from the Central/State Government for the purpose of distributing free food.The Institution/Organization blacklisted under the provisions of Foreign Contribution Regulation Act (FCRA) or under the provisions of any Act/Rules of the Central/State shall not be eligible for financial assistance under the scheme.

Source: https://vajiramias.com/current-affairs/seva-bhoj-yojana/5d2d65571d5def60843d87c5/

Q83. Which of the following companies launched Internet Saathi programme, to improve digital literacy among women in rural areas in India, in order to bring them online? a. Google b. Microsoft c. Amazon d. Facebook

Answer: a

Explanation:

Google India’s ‘Internet Saathi’ programme was launched as a pilot project in 2015. It was launched by Google India along with Tata Trusts. It aims to educate rural women on how to use the Internet. These women, in turn, impart training to other women in their community and neighbouring villages. Hence, option (a) is the correct answer. Source: https://vajiramias.com/current-affairs/internet-saathi/5d2ec1101d5def6080181720/

Q84. Consider the following statements:

1. Its population is confined mostly to Rajasthan and Gujarat. 2. It is listed as “critically endangered” on the IUCN Red List. 3. They generally favour flat open landscapes with minimal visual obstruction and disturbance, therefore adapt well in grasslands 4. It is the state bird of Rajasthan.

Above statements describe which of the following birds? a. Great Indian Bustard b. Northern goshawk c. Great hornbill d. Greater flamingo

Answer: a

Explanation:

The great Indian bustard is moving towards extinction due to several threats. Historically, the great Indian bustard was distributed throughout Western India, spanning 11 states, as well as parts of Pakistan. Its stronghold was once the Thar desert in the north-west and the Deccan plateau of the peninsula. Today, its population is confined mostly to Rajasthan and Gujarat. Small population occur in Maharashtra, Karnataka and Andhra Pradesh. Bustards generally favour flat open landscapes with minimal visual obstruction and disturbance, therefore adapt well in grasslands. In the non-breeding season they frequent wide agro-grass scrub landscapes. They avoid grasses taller than themselves and dense scrub like thickets. These birds are opportunist eaters. Their diet ranges widely depending on the seasonal availability of food. It is the State bird of Rajasthan. It is listed as Critically Endangered on the IUCN Red List. Rajasthan announces Project Great Indian Bustard. They plan to constitute enclosures and secure inviolate areas to ensure successful breeding of birds in the Desert National Park. Hence, option (a) is the correct answer.

Source: https://vajiramias.com/current-affairs/lesser-florican-great-indian- bustard/5d2eb0ce1d5def608c389f6c/

Q85. Markandeshwar temple in Maharashtra is known as the “Khajuraho of Vidarbha”. It is on the bank of which of the following rivers? a. Tungabhadra b. Krishna c. Bhima d. Wainganga

Answer: d Explanation:

The temple of Markandadeo is known as the “Khajuraho of Vidarbha”. It is situated on the bank of River Wainganga in district Gadchiroli of Maharashtra. The temples belong to the Nagara group of temples of North India. Most of the temples have a simple plan, with ardhamandapa, mandapa, antarala and garbhagriha forming the component of the entire set up. On stylistic grounds, their date ranges in between 9-12th centuries CE. The temples belong to Shaiva, Vaishnava and Shakta faith. About 200 years ago the shikhara of the main shrine and maha-mandapa was struck by lightning which led to the partial collapse of the shikhara. The Archaeological Survey of India initiated the largescale conservation work of this temple in 2017. Hence, option (d) is the correct answer.

Source: https://vajiramias.com/current-affairs/markandeshwar- temple/5d2ead941d5def6089ba1586/

Q86. Which one of the following is an artificial ? a. Veeranam lake (Tamil Nadu) b. lake (Kerala) c. (Jammu & Kashmir) d. (Himachal Pradesh)

Answer: a

Explanation:

Veeranam lake turns a major water supplier for Chennai. The Veeranam lake that was dug more than 1,000 years ago by the Chola kings in the then South Arcot district of Tamil Nadu has become a saviour for the city.Hence, option (a) is the correct answer.

Value Addition Vembanad is the longest lake in India,and the largest lake in the state of Kerala. Wular Lake (also spelt Wullar) is one of the largest fresh water in Asia. It is sited in Bandipora district in Jammu and Kashmir. Renuka lake is in the Sirmaur district of Himachal Pradesh. Source: https://vajiramias.com/current-affairs/indias-ongoing-water- crisis/5d2eaebc1d5def6086fc1084/

Q87. What is ploonet, recently seen in the news? a. It forms when moons orbiting a planet escape their orbit. b. It is a Comet Interceptor mission. c. Artificial satellites of the Earth. d. C-type asteroids.

Answer: a

Explanation: Some moons around distant, giant planets can break free of their orbits and end up circling the host star as a planet instead. An international team of astronomers have recently descried these hypothetical moons as ploonets. Ploonets form when moons orbiting a planet escape their orbit. They are flung into a wider orbit around the host star and end up circling like a planet instead. Researchers reveal that ploonets can form when gas giants like Jupiter are forced to migrate closer to their star, causing a transfer of angular momentum to their moons. Such exomoons — moons in other star systems — have around a 50 per cent chance of becoming a ploonet, else they get ejected into space or collide with their planet. No longer shielded by the magnetic field of their original host, however, ploonets are fated for doom — as they are gradually eroded away in the glare of stellar radiation. Hence, option (a) is the correct answer.

Source: https://vajiramias.com/current-affairs/ploonet/5d3016621d5def60801841f4/

Q88. Which of the following is the correct eligibility condition for the ‘Atal Bimit Vyakti Kalyan Yojana’ (ABVKY)? a. The Insured Person should not have been rendered unemployed during the period the relief is claimed. b. The Insured Person should have been in insurable employment for a minimum period of two years. c. The Insured Person should have contributed not less than 90 days during each of the preceding four contribution periods. d. The contingency of unemployment should have been as a result of any punishment for misconduct or superannuation or voluntary retirement.

Answer: b Explanation:

‘Atal Bimit Vyakti Kalyan Yojana’ (ABVKY) is implemented by Employees’ State Insurance Corporation (ESIC). The Scheme has been made effective from 01-07-2018. The scheme is implemented on a pilot basis for a period of two years initially. In case the Insured Person (IP) is rendered unemployed, the scheme provides relief to the extent of 25% of the average per day earning during the previous four contribution periods to be paid up to maximum 90 days of unemployment once in the lifetime of the IP.

The eligibility conditions of the scheme:

• The Insured Person should have been rendered unemployed during the period the relief is claimed. • The Insured Person should have been in insurable employment for a minimum period of two years. • The Insured Person should have contributed not less than 78 days during each of the preceding four contribution periods. • The contingency of unemployment should not have been as a result of any punishment for misconduct or superannuation or voluntary retirement.

Hence option (b) is correct.

Source: https://vajiramias.com/current-affairs/atal-bimit-vyakti-kalyan-yojana- abvky/5d3006071d5def60817e6895/

Q89. With respect to “Solar Charkha Mission”, consider the following statements:

1. Its objective is to generate direct employment for nearly one lakh persons in 50 Solar Charkha Clusters across the country. 2. It is implemented by the National Institute of Solar Energy.

Which of the statements given above is/are correct? a. 1 only b. 2 only c. Both 1 and 2 d. Neither 1 nor 2

Answer: a

Explanation:

Statement 1 is correct: The Ministry of MSME has launched the Mission Solar Charkha for implementation of 50 Solar Charkha Clusters across the country with a budget of Rs. 550 crore. Solar Charkha units have been classified as Village Industries. It has been launched for the year 2018-19 and 2019-20. The objective of the scheme is to generate direct employment for nearly one lakh persons. Statement 2 is incorrect: The Solar Charkha Mission is a Ministry of Micro Small & Medium Enterprises (MSME) initiative launched in 2018. The Khadi and Village Industries Commission (KVIC) is the implementing agency for the programme.

Source: https://vajiramias.com/current-affairs/solar-charkha-mission/5d3130b71d5def45db0ea03a/

Q90. What is Project Sampark, recently seen in the news? a. It was raised in 1975 due to ever-increasing workload of Border Roads Organisation (BRO) in Jammu region. b. It was raised by Assam rifles on June 1990 at Doomdooma Assam to undertake construction activities in the northeastern region. c. It has been launched to reach out to all those who are Left to Follow Up and are to be brought under Antiretroviral therapy (ART) services. d. None of the above

Answer: a

Explanation:

Recently Union Defence Minister inaugurated Ujh and Basantar Bridges in Jammu &Kashmir. The One- kilometre long Ujh bridge is located in Kathua district of Jammu &Kashmir. The 617 Metre long Basantar bridge is located in Samba district of Jammu &Kashmir. Both these bridges have been constructed by Border Roads Organisation (BRO) under 69 RCC/13 BRTF of Project Sampark. The project was raised by Border Roads Organisation (BRO) in 1975 with its HQ at Jammu. It has an area of responsibility from Pir Panjal Range in the north to Pathankot in the south and from Poonch in the west to Dalhousie in the east, covering approximately 2200 km of the road network. Hence, option (a) is the correct answer.

Value Addition Project Udayank: It was raised on June 1990 at Doomdooma Assam by the Border Roads Organisation. It consists of two border road taskforces undertaking construction activities in the northeastern region. Mission Sampark: It has been launched to reach out to all those who are Left to Follow Up and are to be brought under Antiretroviral therapy (ART) services.

Source: https://vajiramias.com/current-affairs/project-sampark/5d3405961d5def503da2d00c/

Q91. Which of the following is the only state in India having all three species of crocodiles; gharial, mugger and saltwater crocodiles? a. Odisha b. Andhra Pradesh c. Bihar d. Gujarat

Answer: a Explanation:

Odisha is the only state in India having all three species — gharial, mugger and saltwater crocodile.Chambal supports the largest population of Gharials in the wild. A gharial has a very long and narrow snout (instead of a broad snout). They are a fresh-water crocodile. They live in deep fast-flowing rivers. Globally Gharial is found only in India and Nepal. It is listed as critically endangered in IUCN Red List. Hence, option (a) is the correct answer.

Source: https://vajiramias.com/current-affairs/gharial/5d3693ad1d5def5049d95d71/

Q92. With reference to the phenomenon of Paid News, consider the following statements:

1. It is any news appearing in print or electronic media for consideration in cash or kind. 2. It has been included in the category of corrupt practices or electoral offences.

Which of the statements given above is/are correct? a. 1 only b. 2 only c. Both 1 and 2 d. Neither 1 nor 2

Answer: a

Explanation:

Statement 1 is correct:

Paid news is defined as any news or analysis appearing in print or electronic media for consideration in cash or kind. Manifestations of paid news are: Advertisements camouflaged as news, Denial of coverage to select electoral candidates, exchanging of advertisement space for equity stakes between media houses and corporate.

Statement 2 is incorrect:

The Press Council of India has received 58 cases of paid news during 2018-19 from the Election Commission and others. No specific instance of paid news in electronic media (Private Satellite TV Channels) has been brought to the notice of his ministry. The Press Council of India has recommended an amendment to the Representation of the People Act, 1951 in order to make the incidence of paid news punishable electoral malpractice. The Election Commission has also recommended for including paid news in the category of corrupt practices or electoral offences.

Source: https://vajiramias.com/current-affairs/paid-news/5d36a60f1d5def5049d95f8c/

Q93. What is Bhabha Kavach, recently seen in the news? a. India’s lightest bullet-proof jacket. b. A medicinal plant that is endemic to the Agasthya hills. c. Bilateral exercise between Indian and French Air Force. d. An active composite volcano on the east coast of Indonesia.

Answer: a

Explanation:

Bhabha Kavach, billed as “India’s lightest bullet-proof jacket”, was recently launched at the International Police Expo 2019 in New Delhi. Hence, option (a) is the correct answer.

The bullet-proof jacket has been developed jointly by the Ordnance Factories Board and the public sector metals and metal alloys manufacturer MIDHANI. The Kavach weighs 9.2 kg, a half kilogram less than the weight for a bullet-proof jacket prescribed by the Union Ministry of Home Affairs (MHA). It can withstand bullets from an AK-47 assault rifle (7.62 mm hard steel core bullets), and the 5.56 mm INSAS rifle. The jacket “is powered with nano technology from Bhabha Atomic Research Centre”, and has a five-year warranty.

Source: https://vajiramias.com/current-affairs/bhabha-kavach/5d368b9f1d5def503f563c82/

Q94. With respect to “Rashtriya Aajeevika Mission”, consider the following statements:

1. Its objective is to organize the rural poor women into Self Help Groups (SHGs), and continuously nurturing and supporting them to take economic activities. 2. It is being implemented only in Aspirational Districts by the Ministry of Rural Development.

Which of the statements given above is/are correct? a. 1 only b. 2 only c. Both 1 and 2 d. Neither 1 nor 2

Answer: a

Explanation:

Statement 2 is incorrect: Deendayal Antyodaya Yojana – National Rural Livelihoods Mission (DAY-NRLM) is implemented by the Ministry of Rural Development. It is being implemented across the country in a mission mode.

Statement 1 is correct: The Mission objective is to organize the rural poor women into Self Help Groups (SHGs), and continuously nurturing and supporting them to take economic activities so that they come out of abject poverty. Sub-schemes under DAY-NRLM:

• Rural Self Employment Training Institutes (RSETIs), enables a trainee to take bank credit and start his/her own Micro-enterprise. • Start-up Village Entrepreneurship Programme (SVEP) for facilitating the rural poor for setting up of micro-enterprises. • Aajeevika Grameen Express Yojana (AGEY) to facilitate transport facilities in the rural areas which also provides self employment opportunities to the rural poor.

Source: https://vajiramias.com/current-affairs/rashtriya-aajeevika- mission/5d37ec2a1d5def504107bafb/

Q95. Which of the following are the steps taken by India to combat terror financing in the country?

1. A Terror Funding and Fake Currency (TFFC) Cell has been constituted in the National Investigation Agency (NIA). 2. FICN Coordination Group (FCORD) has been formed by the Ministry of Home Affairs. 3. The recent ratification of the United Nation’s Comprehensive Convention on International Terrorism (CCIT).

Select the correct answer using the code given below: a. 1 only b. 1 and 2 only c. 2 and 3 only d. 1, 2 and 3

Answer: b

Explanation:

The steps taken by India to combat terror financing in the country:

• Strengthening the provisions in the Unlawful Activities (Prevention) Act, 1967 by criminalizing the production or smuggling or circulation of high quality counterfeit Indian currency. • A Terror Funding and Fake Currency (TFFC) Cell has been constituted in the National Investigation Agency (NIA) to conduct focused investigation of terror funding and fake currency cases. • FICN Coordination Group (FCORD) has been formed by the Ministry of Home Affairs to share intelligence/information among the security agencies of the states/centre to counter the problem of circulation of fake currency notes. • A Memorandum of Understanding (MoU) has been signed between India and Bangladesh to prevent and counter-smuggling and circulation of fake currency notes.

India proposed a draft document on the Comprehensive Convention on International Terrorism (CCIT) at the UN in 1996 but it has not been implemented as there is no unanimity on the definition of terrorism among the member states. Although consensus eludes towards adoption of the terrorism convention, but discussions have yielded three separate protocols that aim to tackle terrorism: International Convention for the Suppression of Terrorist Bombings, adopted on 15 December 1997; International Convention for the Suppression of the Financing of Terrorism, adopted on 9 December 1999; and International Convention for the Suppression of Acts of Nuclear Terrorism, adopted on 13 April 2005.

Hence, option (b) is the correct answer.

Source: https://vajiramias.com/current-affairs/terror-financing/5d37e51e1d5def504107ba16/

Q96. According to a Reserve Bank of India, a default is wilful when it fulfils which of the following conditions?

1. A borrower (or an entity) does not pay up even when it has the capacity to pay.

2. A borrower sells assets given as security against the loan without informing lenders.

3. A borrower has not used the loan for the purpose borrowed and diverted it elsewhere.

Select the correct answer using the code given below: a. 1 only b. 1 and 2 only c. 2 and 3 only d. 1, 2 and 3

Answer: d

Explanation:

According to a Reserve Bank of India, a default is wilful when it fulfils one of the following four conditions:

• A borrower (or an entity) does not pay up even when it has the capacity to pay, • a borrower has not used the loan for the purpose borrowed and diverted it elsewhere, • a borrower has siphoned off the funds and the money is not available with it in the form of other assets, and, • a borrower sells assets given as security against the loan without informing lenders.

Hence, option (d) is the correct answer.

Source: https://vajiramias.com/current-affairs/wilful-defaults/5d37e7f41d5def50457736ff/

Q97. What is Tiangong-2, recently seen in news? a. A manned Chinese space station. b. An application of block chain technology. c. An artificial Satellite launched by Space X. d. Electric plane tested by Japan.

Answer: a

Explanation:

Tiangong-2, a manned Chinese space station, was launched on September 15, 2016 and, in late 2016. It hosted two Chinese astronauts for 30 days in what was China’s longest manned space mission so far. Tiangong-2 was retired from service after it had completed its experiments in space. Most of Tiangong-2 burnt up in the atmosphere, and the remaining debris fell near Point Nemo, the most remote location on Earth. Hence, option (a) is the correct answer.

Value Addition – Spacecraft cemetery / Point Nemo ‘Oceanic pole of inaccessibility’ is the place in the ocean that is farthest from land. It lies in the South Pacific Ocean. The nearest land is approximately 2,415 kilometres away. It is also Known as ‘Point Nemo’ in Latin Nemo means ‘no one’. Point Nemo is relatively lifeless; its location within the South Pacific Gyre blocks nutrients from reaching the area, and being so far from land it gets little nutrient run-off from coastal waters. The area is also known as a ‘spacecraft cemetery’ because hundreds of decommissioned satellites, space stations, and other spacecraft have been deposited there upon re-entering the atmosphere. It has been chosen for its remoteness, so as not to endanger or harm human and oceanic life. Source: https://vajiramias.com/current-affairs/tiangong-2/5d37f2401d5def5040c56cae/

Q98. India is a signatory to which of the following United Nations (UN) Conventions?

1. UN Refugee Convention of 1951. 2. United Nations Convention against Transnational Organised Crime (UNTOC). 3. Universal Declaration of Human Rights, 1948.

Select the correct answer using the code given below: a. 1 and 3 only b. 1 and 2 only c. 2 and 3 only d. 1, 2 and 3

Answer: c

Explanation:

UN Convention relating to the Status of Refugees: It is also known as the 1951 Refugee Convention.The convention was approved in a UN conference in 1951 and came into force in 1954. This convention defines who is a refugee and which people do not qualify as refugees; Sets out the rights of individuals who are granted asylum and sets out responsibilities of nations that grant asylum. India is not a signatory to the UN Refugee Convention of 1951 or the 1967 Protocol, which protects refugee rights.

In May 2011, the Indian Government ratified two UN Conventions - the United Nations Convention against Corruption (UNCAC) and the United Nations Convention against Transnational Organised Crime (UNTOC) and its three protocols. Universal Declaration of Human Rights, 1948: Article 14 of the 1948 Universal Declaration of Human Rights recognizes the right of persons to seek asylum from persecution in other countries. India played a crucial role in the official adoption of the Universal Declaration of Human Rights (UDHR) – characterised as one of the world’s greatest living documents, as the ‘common language of humanity’, ‘the conscience of the world’. Hence, option (c) is the correct answer.

Source: https://vajiramias.com/current-affairs/franco-german-migration-resettlement- plan/5d37f7181d5def504107bc61/

Value Addition – Refugee crises may be caused by any number of reasons but the most common are war (Bangladesh), domestic conflicts (Tibet, Sri Lanka), natural disasters (famine), environmental displacement, human trafficking and climate change. India has signed neither the 1951 United Nations Refugee Convention nor its 1967 Protocol, which has 140 signatories, an overwhelming majority of the world’s 190-odd nations. However, India continues to host a large population of refugees. Why won’t India sign the Convention or the Protocol? First reason is that the borders in South Asia are extremely porous and any conflict can result in a mass movement of people. This can have two results: first, a strain on local infrastructure and resources in countries that are poorly equipped to deal with sudden spikes in population. Second, it can upset the demographic balance, a tinderbox in South Asia. Secondly, India already does its duty, so there is no need to sign this piece of paper. A third reason offered by some scholars is that India retains a degree of scepticism about the UNHCR which flows from the Bangladesh war of 1971. India mostly doesn’t take UN money to look after the refugees. According to the UNHCR, there were 204,600 refugees, asylum seekers and “others of concern" in India in 2011. They were made up of 13,200 people from Afghanistan, 16,300 from Myanmar, 2,100 from various other countries and the two older populations of around 100,000 Tibetans and 73,000 Sri Lankan Tamils. The UNHCR financially assisted 31,600 of them.

Q99. With reference to the Dracaena cambodiana, consider the following statements:

1. It is a recently discovered dragon tree species in Goa. 2. It is for the first time that a dragon tree species has been reported from India.

Which of the statements given above is/are correct? a. 1 only b. 2 only c. Both 1 and 2 d. Neither 1 nor 2

Answer: b

Explanation:

Statement 1 is correct: Researchers have discovered Dracaena cambodiana, a dragon tree species in Assam. Researchers have discovered Dracaena cambodiana, a dragon tree species in the Dongka Sarpo area of West Karbi Anglong in Assam. This plant yields dragon’s blood — a bright red resin used since ancient times as medicine, body oil, varnish, incense and dye. Sap of this plant turns bright red after coming in contact with air.

Statement 2 is incorrect:

This is for the first time that a dragon tree species has been reported from India. The report has been published in the Journal of the Botanical Research Institute of Texas. In India, the Dracaena genus belonging to the family Asparagaceae is represented by nine species and two varieties in the Himalayan region, the northeast and Andaman and Nicobar Islands. But Dracaena cambodiana is the only true dragon tree species. Recent overexploitation to meet the increasing demand for dragon’s blood has resulted in rapid depletion of the plant. For this reason, the species is already listed in the inventory of Rare and Endangered Plants of China.

Source: https://vajiramias.com/current-affairs/dracaena-cambodiana/5d39521c1d5def7124b052fb/

Q100. The Fair and Remunerative Price (FRP) of Sugarcane is approved by the: a. Cabinet Committee on Economic Affairs (CCEA) b. Commission for Agricultural Costs and Prices (CACP) c. Directorate of Marketing and Inspection, Ministry of Agriculture d. Agricultural Produce Marketing Committee (APMC)

Answer: a

Explanation:

Price of sugarcane is fixed by the centre/State, while the price of sugar is market determined. Fair and remunerative price (FRP) is the minimum price at which rate sugarcane is to be purchased by sugar mills from farmers. The FRP is fixed by Cabinet Committee on Economic Affairs on the basis of recommendations of Commission for Agricultural Costs and Prices (CACP). The ‘FRP’ of sugarcane is determined under Sugarcane (Control) Order, 1966. Recommended FRP is arrived at by taking into account various factors (cost of production, demand-supply situation, domestic & international prices, inter-crop price parity etc. Hence, option (a) is the correct answer.

Source: https://vajiramias.com/current-affairs/fair-and-remunerative-price- frp/5d3941e91d5def7126ed3112/